Abnormal Uterine Bleeding Would you be able to obtain repeat endometrial bx after inserting IUD Slide 18 Yes How long would would you treat AUB with medroxy-progesterone, also when stopping the therapy does it need to be tapered Clinically - I treat for three months. The data is VERY limited on duration. choice for young women with hirsutism who do not want to become pregnant.

Spironolactone, in daily doses of 50-200 mg, blocks androgen receptors. Spironolactone also decreases testosterone production, making it additionally effective for hirsutism.

Sexually active women taking spironolactone should ensure that contraceptive measures are adequate. In some cases, spironolactone can be combined with an OC for added effect on the hirsutism.

With current systemic therapies for hirsutism, 6 months to a year of therapy is usually required before results are noticeable. Even then, only approximately one half to three quarters of patients show improvement. The problem may lie partially in the nature of the , which persists for 6 months to a year even after androgen levels have been normalized. Ineffectiveness may also be due to the inability of treatment to completely normalize elevated tissue dihydrotestosterone levels. Newer therapies directed at inhibition of 5-alpha-reductase or blockade of the androgen receptor may improve the ability to treat what is the dose for spironolactone for hirsutism patients.

Acute and Chronic Cognitive Diseases

Common Issues in the Elderly I would be most appropriate because they wouldn't be rushed to further intervention. Certainly, if there are other red flags on exam or sx don't improve with conservative management, imaging is an appropriate next For lumbar stenosis, why would you not do an MRI first step is post prandial hypotension common in younger people too Not as common because younger people typically have better overall flow For lumbar stenosis, what if conservative therapy (i.e. PT, medications) does not work Then MRI, etc and further is very appropriate Big one for clozapine is severe neutropenia. All have some anticholinergic effect and increased fall risk. Quetiapine and clozapine have lower risk of extrapyramidal sx compared to others, but that risk still isn't Side effect of clozapine and quetiapine that we need to know for test zero.

Common Issues in the Elderly II Not necessarily. BUT...paroxetine specifically is highly anticholinergic and shouldn't be used in elderly Are SSRI's associated with developing dementia patients what about the Frax score When do we need that on the board exam It's worth knowing about and reviewing the site for sure. I wouldn't worry about intricate details regarding calcitonin not useful for pain in acute hip fx, only vertebral compression Also helpful for any osteoporotic fx actually How much do OAB meds contribute to dementia (central anti-cholinergic effects) Emerging data that it raises the risk, but this is also all still fairly early data it's effective in decreasing fx. Also remember if someone is on osteo treatment they need to be on cavit d what is the data for ca and vid supplementation benefit for patients already have osteo too Even if there is >100,000 colonies of whichever , on urine Cx, still not treating if Correct. Colonization is pretty common as we age and our treatment with abx causes more harm than asymptomatic good if no sx. What is your approach with UTI treatment in patients with intellectual disability who can't communicate effectively Good physical exam and sorting through new urinary sx (frequency, difficulty, behavior changes, etc). when treating for RLS w sinimet, when would you recommend stopping & what potential risks As they get older and from time to time, I would try to wean it. Side effects and risk are related to would you be most concerned about dopaminergic activity. Also some rare cardiac risks with sinimet Not necessarily. Based on risk factors, it's still reasonable to screen a male (i.e. on chronic steroid therapy, Are the only men that need screening with DXA age >50 and a fracture long term PPI, etc) Do we have data on falls secondary to medication side effects (i.e. how many femoral neck fractures do WE cause) I can't give specifics, but there is definitely good data out there, particularly related to falls in general What to do when pt having cognitive decline, no family involvement, and refuses home health or This could be a situation for APS or some other entity to come in and evaluate. But there are no great moving into assisted living options and it will be messy USPSTF does not recommend to check BMD for pt receiving RX for osteoporosis, then how do we assess the efficacy that we need to change rx or not If you think it will change management, certainly getting a dexa is appropriate just want to clarify if pt is going into hospice.. stopping statin (with hx of CVD) would increase per the data, yes. But worth discussing with the patientfamily for shared decision making. In an elderly that cannot swallow appropriately (I.e. stroke) but was previously able to swallow, They would be appropriate for PEG potentially. The recommendation against feeding tube is for advanced what are the options for feeding dementia patients. Would you perform BMD testing in an African-American male with hip fracture I would treat anyway but probably would to get a baseline why stop biphosphanate after 5yrs tx side effects go up the longer someone is on Do Patients taking Biphosphonates should DC it for tooth extraction I would as the risk for jaw necrosis goes up with invasive dental procedures how often do you check vitamin dcalcium levels I only check vit D if there is a fracture or someone is osteoporotic on DXA. That helps me choose vit d Do you need to obtain a DXA in postmenopausal woman after a hip fracture or can you start treatment for osteoporosis You can start. However, it would be reasonable to get a DXA for bl You could, but they're not going to move the needle. And potentially cause more harm than good if the Why can't you use high-calorie supplements in older, demented pts who are not eating patient shows no interest in food. No great guidelines on this. Hopefully more to come. There are bisphosphonate dosing options for How do we treat osteopenia based on new guidelines osteopenic patients. I have not seen gastric bypass as a contraindication for oral biphosphonates. Since pt unable to drink full glass of water and risk of reflux is greater, What is your opinion definitely a pt that either iv bisphosphonate or prolia would be better choices Lots of evidence for Vit D decreasing fall risk in nursing home patients. This isn't true for community What studyevidence is there for Vitamin D in nursing home patients dwelling adults The challenge is if the person takes a po bisphosphonate and synthroid. You'd have to find a regimen that is consistent and doesn't overlap so both could be taken on "empty" stomachs. IV bisphosphonate would What about with hypothyroid patients how often be reasonable if approved by insurance too. what is the next step if pt is on treatment and gets a compression fx I would switch to an agent with different mechanism probably. So Cal-VitD not for primary prevention, but ok for pt with Osteoporosis, right correct when would you use z score in BMD result Probably doesn't affect treatment decisions. Mainly t score

Diabetes They are 3rd line. metformin 1st line 2nd line is GLP1SGLT2 if you can afford it, sulfonylurea or pioglitazone if you can't Metformin, then GLP1SGLT2. You said then sulfonylureas. Where do DPP4 meds come in DPP-4s are more expensive but not as good at lowering A1c as GLP1 or SGLT2 lantus vs toujeo.. is there a difference other than higher concentration is toujeo better for CKD They are the same thing, just a different concentration So, SGLT 2 inhibitors should not be used with patients already with peripheral neuropathy No, they can be used if not screening for microalb, how to know of CKD that is a concern for meds. Theoretically they are on the correct medicine Hmmm, not sure of the question, but ACEARB are the drugs of choice for hypertension with ARB better ACEARB in Dm tolerated. Continue one of them if you can. Don't give both. Reglan versus for gastroparesis For the test it is Reglan. Erythromycin is also fine 30-300 mgdL day of albumin in the urine. We usually just do a microalbumincreatinine ratio. If this is elevated, then we consider it to be microalbuminuria. Start treatment when it is positive or in the how diagnose for microalbuminuria and when do you start treatment presence of hypertension since we are using the same drugs. Asian Americans it says bmi of 24 Earlier talk said bmi less than or equal to 23. Which is correct they tend to develop diabetes at 24. Recommend screening at 23 The comment on statins over age 10 on the treatment in children slide - is that only if lipids indicate treatment yes, thank you so we do not focus of Cr when it comes to metformin test should not ask Yes, they will ask about GFR Are the diabetes medications on boards name brand, generic or both They will give you generics for the old ones and both for the new ones Why do we avoid stoping metformin with sulfonyurea Increases morbidity Should DPP-4 Inhibitors be avoided in patients with inflammatory and osteoarthritis because of the side effect of joint pain No it is very rare, but just be aware of it if they develop new joint pain, they should come off If over age 70 and on testosterone therapy, is screening PSA recommended NO How often should you screen for thyroid disease and celiac disease in children with diabetes Usually just after diagnosis and then if you have any indications If a patients A1c is controlled and in non-diabetic range would you still recommend keeping metformin on bc of benefits YES!!!! at what level of renal failure would you give Prevnar The recommendation is just "chronic renal failure". Definitely by time they get to class III At what point do we start tapering off diabetic medication in our patients Are there guidelines for No guidelines. Would generally continue metformin unless the A1c is really low < 5.7. A1c may start to rise this again if they gain weight or just as they get older so I generally just leave them on 500 mg daily < 14090 BP goal for diabetics for test unless they specifically ask about the American Diabetes Association and then it is 13080 Not necessary to make diagnosis but Beta-hydroxybutyrate is a better measurement of the degree of for DKA if ketone in urine is noted do you need a serum ketone to make dx ketosis than serum ketones Would you add Diltiazem for a patient who has persistent microalbuminuria despite maximal doses of ACE or ARB For blood pressure but not just for the microalbuminuria although they usually go hand in hand Do you recommend blood sugar checks if the patient is not on insulin but is on oral For sulfonylureas and meglitinides Never take pt off Metformin even if they are consistently below A1C 5.6 Individualized decision and watch for future increases in A1c You spoke about starting Metformin 500mg PO q daily and increasing as needed. Are you referring Not sure I understand the question, but if someone has an elevated A1c I start metformin at 500 mg, if the to the ER for or immediate A1c is not < 6.5, I will increase metformin slowly to 2 gday until A1c is < 6.5 or I have to add another drug Stop sulfonylurea or meglitinide when you start insulin Don't stop metformin Probably don't stop SGLT2 or GLP1 unless you need to When do you stop oral diabetic medications after starting insulin Everything else, you just have to decide whether you are getting a benefit or not and what the cost is Every 2-3 years if normal Every year if it is found to be low to monitor supplementation How often does B12 need to be monitored on Metformin There is no standard recommendation Yes, > 40 you would use high dose statins (< 40 if high risk for CVD) if LDL is > 70. See slides at end of are statins recommended for women w diabetes and ldl <100, no other risk factors lecture and there will be a lecture on lipids Have you seen acute onset Type II diabetes caused by a viral illness No, although you could get type 1 12. If that is the case they will move fairly quickly to needing insulin

Health Promotion and Prevention The Men-B is a one time vaccine. Any evidence for an additional MEN-B in a child who already completed the series before going off Persons 16-23 years of age may be vaccinated to provide short-term protection against most strains of to.college meningococcal B disease Pneumovax 23 or Prevnar 13 in pregnancy Both are acceptable GSK is continuing to steadily increase supply of Shingrix for the US market and anticipates producing and distributing more doses in 2020 than in 2019. Due to high levels of demand for Shingrix, GSK will continue to actively manage volume and timing of shipments to ensure distribution on a consistent and predictable schedule across customer channels. While order limits will remain in effect for the near-term, most providers can expect to see the same number or an increase in the number of doses available in 2020 and Any news when new shingles vaccine will be in supply again shorter wait times for delivery. (This is the most recent CDC statement - December 2019) Fatty liver give single dose or 3 shot series for Hep B Full series. Is there generally insurance coverage for HPV immunization in people 25-47 years of age Yes - as of August 2019. Women who have close blood relatives with breast cancer have a higher risk

Having a first-degree relative (mother, sister, or daughter) with breast cancer almost doubles a woman’s risk. Having 2 first-degree relatives increases her risk about 3-fold. Women with a father or brother who has had breast cancer also have a higher risk of breast cancer. Overall, about 15% of women with breast cancer have a family member with this disease.

About 5% to 10% of breast cancer cases are thought to be hereditary (genetic), meaning that they result directly from mutations passed on from a parent.

BRCA1 and BRCA2 The most common cause of hereditary breast cancer is an inherited mutation in the BRCA1 or BRCA2 gene. In normal cells, these help make that repair damaged DNA. Mutated can you please explain the difference between familial and genetic re breast cancer versions of these genes can lead to abnormal cell growth, which can lead to cancer. The USPSTF (August 2018) recommends screening for cervical cancer every 3 years with cervical cytology alone in women aged 21 to 29 years. For women aged 30 to 65 years, the USPSTF recommends screening every 3 years with cervical cytology alone, every 5 years with high-risk human papillomavirus (hrHPV) testing alone, or every 5 years with hrHPV testing in combination with cytology (cotesting). Grade A

Evidence from randomized clinical trials (RCTs) and decision modeling studies suggest that screening with cytology alone is slightly less sensitive for detecting CIN 2 and CIN 3 than screening with hrHPV testing alone. Although screening with hrHPV testing alone or in combination with cytology detects more cases of Can you explain the change in Pap recommendation CIN 2 and CIN 3, this method results in more diagnostic colposcopies for each case detected. The simple fact is both caffeine and nicotine are stimulants - they increase your metabolic rate for a period of time. You burn a few more calories than you normally would in the same time frame doing the same things. Stimulants are rarely a good choice though when it comes to losing weight and I would never condone a stimulant program for weight loss. Take in less food and exercise more that is the secret to long term healthy weight loss.

Duration of therapy — In general, pharmacotherapies for smoking cessation are recommended for two to three months. However, NRT may be extended and even used indefinitely if needed. Side effects from using NRT are related to the type of product, and include irritation from patches and irritation to the inside of the mouth from gum and tablets.Th e premise of long-term NRT use is that it would prevent Evidence for use Nicotine Replacement Therapy (NRT) for weight loss health concern for person on smoking relapse, and the risks would be limited to maintaining nicotine addiction, but without the adverse NRT indefinitely consequences of continued smoking, such as increased lung cancer risk and heart disease. Can you use short acting Buproprion in patient with eating disorder Yes.

Endocrine Diseases Certainly at the TSH is greater than 10 and the patient is symptomatic you should treat. It suppressed TSH It TSH is > 10 or < 0.1, should you not treat because of possible effects (osteoporosis, A Fib, CHF to that level will always require treatment. This would include both beta blockers and methimazole. You mentioned things that decrease T4 to T3 conversion. What does this do to TSH If the T4 to T3 conversion is diminished by whatever reason, this will likely increase TSH if the feedback IncreaseDecrease loop is maintained in the HPA axis. should hypothyroid patients completely avoid cauliflowerbroccoli No you can continue to eat cauliflower and broccoli but continue with supplementation. If the patient is stable, I should still check annually. If they are symptomatic then they should be checked more frequently. There’s no great data to say exact frequency of what you said should be checked. The How frequently do you prefer to check TSH on people who are stable on levothyroxine surveillance question will not likely be on the boards. Should methimazole be stopped if they are started on levothyroxine Yes Yes the radioactive islands that will be performed first typically, especially if the TSH is suppressed. There are requirements that describe the size along with the atypical characteristics that would determine cancer. Based on these indications, this helps to determine when you would biopsy. For example, Elysian it For thyroid module work up is coldhot differentiation needed before FNA I thought size of module is less than 1 cm with overall nine concerning features you do not necessarily have to biopsy. These criteria drove FNA decision.. described by the endocrine society. cancer. This patient will most likely need to be on thyroid supplementation for life post thyroidectomy. Therefore, repeat Ultrasounds you will need to pursue routine surveillance depending on the patient’s symptoms. We have like 54 is about seven days. This is why are you connected us once weekly all of this is not the Can you comment about alternating doses of synthroid Is this evidence based community standard or norm. It is the standard to dose daily. This is an interesting question. I’m not sure of the exact answer. In part, this will be dependent on the symptoms of the patient with Graves’ disease. If you’re managing this patient from and obstetrics PTU during 1st tri and then methimazole and then restart ptu until she stops breastfeeding how perspective, you will need expert consultation for this. This type of question will be unlikely to be on the long to wait between stopping PTU and methamazole after she stops breastfeeding board. You can check a urinary cortisol, however it is more routine to check and fasting a.m. cortisol and then to cushing syndrome - we should check urinary cortisol follow up with a dexamethasone suppression test. Patients with Addison’s will likely need increased supplementation if they are in state of crisis such as Addison's disease - stress dose question expected sepsis, or post surgery, or periods of increased metabolic demand. I answer this in previous question. Sometimes even though the TSH is normal, patients can be symptomatic. And other times patient will be completely euthyroid in terms of their symptoms even if they have a normal TSH but are "not feeling well"-- what does that mean do you diagnosed them though their TSH was elevated. This is really an individualized question. Certainly the TSH is greater than 10 as hypothyroid based on symptoms but not the TSH level be very unusual for patients not to be symptomatic. if T4 WNL but TSH borderline elevated with symptoms, should we start treatment Should only start thyroid supplementation if the patient is symptomatic. It really depends if the patient is symptomatic or not. Some patients are still symptomatic even though At what TSH value does one initiate levothyroxine their thyroid, or TSH, is in the normal range. Which thyroid antibodies should I check for hypothyroid -

Obesity and Metabolic Syndrome Your opinion how much of Belviq's increased cancer risk is from the drug itself versus the Likely we are dealing with a system at greater risk because of the obesity and contributed to by the underlying pre-existing obesity (slide 172) lorcaserin why does statin increase risk of DM It's effect on insulin production and secretion The number for 50%-69% of max heart rate seems low for exercise and weight loss. Is this the fat burn range Seems that the fitness range heart rate would be higher success of Fitness range is better, but after 20 minutes of moderate exercise you burn your immediate glucose stores weight loss. What are your thoughts and begin to convert fat--even at that heart rate Do shellfish count in the seafood recommendation yes, just be aware of caloric and cholesterol issues -- so moderation and should include fish if possible There is an effect on gut bacteria (bacteriostatic). Studies in mice--more on the older sweetners. More to come on this. They also have an effect on metabolism. I would prefer not to use them, but so far they Do you have an opinion on artificial sweeteners adversely affecting gut flora have been shown to be relatively safe and are better than all that sugar. Everything in moderation. . . there was a recall of belviq due to increased risk of cancer on feb 13th. will that be on the boards no Well it does effect blood pressure so can affect cardiac status from that. It has not ben shown to cause any Doesn’t phentermine have cardiac & Pulm hypertensive risks increase of pulmonary hypertension Can the "sweetness" of artificial sweeteners still trigger spikes of insulin release leading to Yes, they actually have an effect on GLP-1. Mostly it is believed that they stimulate the taste sensation of resistance So they could still contribute to weight gain & diabetes despite having zero calories. sweet influencing people to eat moe. For BupropionNatrexone for binge eating, I thought Bupropion is contrainidicated for eating It is not really for binge eating disorder (sorry to give that impression--I think it was the gallon of ice cream disorders thing). It just seems to work better for patients who have more cravings. --Not a board question No, you can use it in well controlled hypertension, but you will need to monitor their blood pressure Is hypertension an absolute contraindication for phentermine, even if it is well controlled because sometimes they become not so well controlled etc) I am taking care of all of these by working on the diabetes. It does require that I write something about those other things. I always code for their obesity as well. Also some insurance companies are how do you code for obesity related visits Often insurances wont pay those visits. paying I would know that it interacts with SSRIs. They will likely pull the question, but you should still be able to There's a recent question about use of Belviq. What is your advice answer correctly. LocaserinBelviq, removed from market last week due to increased risk of cancers. Will this be on It will definitely not be on boards--i.e. the part about causing cancer or that it is pulled from the market--far boards to early for them to develop a question about that Does exercise multiple times a week give better results than exercise 2-3 hours weekly in one Actually the data shows that exercise at least 5 times a week is better than one. The "weekend warrior" is setting harder on the body, and regular exercise is better. If pt had gestational diabetes, and glucose normalizes post delivery, do you still start That decision is really up to you and the patient and will not be the same for every patient. You will need metforminacarbose to follow that patient more closely because of their increased risk for developing overt diabetes.

Preoperative Examination and Surgical Management How much sodium bicarb should we mix w local anesthesia to reduce injection site pain 1 part 1% lidocaine to 10 parts 8.4% sodium bicarb. This is the rub because of the volume. The studies I have seen show improvement with both, but there are a lot of studies. Happy to look at the Using LMWH with mechanical pneumatic device is said to increase mortality. Is that true one you are referencing. My email is [email protected]. Thanks How many days on dental prophylaxis if indicated Just day of procedure. Dose about 1 hour prior to procedure Should you stop Methotrexate for procedures other then hip or knee Yes, How do we handle preop clearances when surgeon asks for one prior to a surgery that does might not need a preop clearance (I.e. cataract See the patient and then you can give them clearance Complicated answer because the risk is CVA (risk is about 2%) not MACE. The risk of MACE is probably in the moderate risk area. The SAPPHIRE study recommends a further evaluation strategy for these patients based on 1) age 2) in a nursing home 3) HF 4) COPD 5) Diabetes 6) history of CVA 7) degree of contralateral ICA stenosis Sorry, no one seems to know the real answer here. You will have to make it based on the risk of your Does a carotid endarterectomy count as a moderate risk surgery patient and the above should be helpful what is the latest recommended prophylaxis for DVT prevention after ortho surgery They continue for a month following the surgry Cards told me cont. ASA and not stop even w high risk surgery bc any operation places risk for thrombotic event Thoughts Did the patient have a stent or was significantly high risk How do you discuss with ped parents who smoke that hernia repair cannot happen because of smoke in the home Just if the child shows up with a URI. So they should stop smoking so it won't happen ) Stop based on their half-life 3-5 days before and start usually the night after or the next day (based on Starting and stopping anticoagulants before and after surgery surgery and patient) If diabetic pt is NPO and surgery isn't until later in the day due you put them on NS or D5 I have Usually NS, only D5 if you think they are becoming hypoglycemic. You can always do a fingerstick Instead of an A1C would be reasonable to get a random blood glucose while doing a prep Yes, especially if you already know what their A1c runs What about BNP in patients over 65 years old Only if they have heart failure and you think it is changed or you think they have undiagnosed heart failure If lab tests are needed how current do they need to be from the date of surgery Usually within the last month when could we stop blood thinners before surgery On the slide, it is different for different ones. Usually 3-5 days Well they can, they do it. SO, you can follow the same recommendations and (unless they are high risk-- Dentists always asking for pre-extraction evaluation especially whether or not they are on blood like mechanical heart valves, DVT or stroke within 3 mo, etc) you can stop their anticoagulant just like you thinners. Can they do this would for surgery. That should make them happy What about simple procedures like biopsies or colonoscopies Low risk How many days prep should herbals be stopped Preferably at least a week. They are not needed and will just react with other things is dental proplylaxis indicated for patients with knee replacement No, interestingly not by the orthopedic or dental societies Based primarily on his age. i.e. 7-10 would be a healthy young person who could run a mile -- well 10 Why was case 2 not METS 7-10 would run the marathon That is because it is not really clear. If they have been followed closely and doing well, they likely do not For stable CAD, mets>4, moderate risk surgery, do we need ekg your slides contradicted one need it. Often we get an ECG in case there are changes duringafter the surgery. This is where you have to another use your best judgment. determine patient reliability as well as the other factors you mentioned. This is for a good recovery post op. This seems to be left out of most lectures. ie are they on drugs will they do physical therapy etc.... Yes, well said!! For preop, when is it indicated to get PTPTTINR on top of the standard hemoglobin or CBC History or family history of bleeding disorder Does colonoscopy needs any pre op testing Usually not, it is a low risk procedure Not sure but may have something to do with less optimization and awareness than in the patient where Why is diastolic heart failure more dangerous than systolic in preop risk evaluation the failure is a bit easier to see.

Unique Geriatric Pharmacologic Issues Which Antihistamine do you prefer in elderly I try a nasal spray so it's not systemic first. If you have to use something po, still use lowest effective dose what is the best drug to treat depression in elderly Depends but the wrong answer is paroxetine How to best medically treat insomnia in geriatrics CBT and melatonin. Maybe even lavender. did the fall risk go up in the sprint trial for tighter blood pressure control in elderly patients It did not, at least not statisitcally significantly I would start over in her, based on how well she improves after stopping the other meds. If she completely for the Manny pills lady what would be HTN risk of strokeMI if stop her bp rx. recovers, I'd be more aggressive. Which muscle relaxant did you say to use If you have to, robaxin is safest bc of quick half life

Urologic Problems Can coffee prevent recurrence of kidney stones How much Mixed data. Depends in part on hydration status. 1000 Psas 240 positives 100 get a bx 80 get surgery 1 person avoids death 3 avoid cancer from spreading when DO you check PSA 5 die from cancer even post surgery or tx In obstructive uropathy with AKI, is giving bolus of IVF put a patient at an increased risk of developing hydroureter Potentially. If obstruction. Need urology for decompression. For confirmation, are Alpha blockers beneficial or not reagrding management of stones Alpha blockers helpful in BPH, but don’t really affect stone formation. If you have an incidental kidney stone (say, 5mm or greater) found on CT scan, do we need to send to Urology to have it removed No, can wait.5050 chance it will pass. what are the familial and genetic risks for primary hypogonadism beyond Kilinefelters etc Best to review that slide for the secondary causes. What place does clomiphene have in the treatment of testosterone deficiency in men Used rarely. Can Buproprion be used for ED No good data. What are the major contraindications to using PDE5-is Hypotension. Although not recommended, can you still use nitrates with sildenafil Or is this primary a This puts you at risk for hypotension. Can eliminate the Test tx because they are the same. In general the CAM answers which has decreased Question #5...Please expand on the comment, “look at how the question is designed”. data are less likely to be answers. Is there utility to getting CT in patient with known history of stones without evidence of obstruction or infection Can prob avoid if asx. Can pursue X-ray for surveillance if you know it is there. Do you need to do DEXA scan for men with hypogonadism Yes should consider. Does nifedipine & flomax help pass kidney stone or not Mixed data but newest data shows some potential benefit. You've identified symptomatic low testosterone. Do you work up everyone for causal factors No need to do exhaustive work up unless other so that could implicate pituitary disorder. If fulminantly sx beyond primary low T ok to treat if no risk factors. what about the young men with low T. what are the causes of this seemingly epidemic Prob diet, exercise,obesity.

Welcome and Overview You have access to the previously recorded nn-line course. The online course is linked on the Houston Live Course Home Page. The link is entitled Access Board Review Central.

Can we get access to the videos of these lectures after the course (How) Houston Live Course Home Page www.aafp.orghouston-info

ACS and Hyperlipidemia could statin therapy be intermittent (take holidays off the drug) if side effects arise You can give a statin holiday, then restart at a lower dose. Treating hyperlipidemia in adolescents. Screening starts at age 40, unless history of familial hypercholesterolemia. Can you use the CK-MB to see if it is a new MI Yes. This is about the only indication for it, but it can be uniquely helpful in this circumstance. There is a theory that if the beta receptors are blocked, it exposes only the alpha blockers to activation by Why do you need to avoid beta blockers for cocaine-induced ACS What is the consequence cocainemetabolites, leading to a hypertensive crisis. Within 12 hours is the guideline of use for rural hospitals, to help drive decisions regarding fibrinolysis on Reperfusion win 12 hrs - we were always taught “door to balloonstent time of =< 90 minutes”. Is site versus transfer to a PCI-capable hospital. If the patient is at a PCI-capable hospital, the sooner the that not the case anymore better. Many cath labs are proximal to the ED for this reason. will the abfm test using the ascvd risk of 10% or 7.5% indication for starting high dose statin The board will not ask questions in the "gray zone." Would I be wrong to stop all statins in elderly >75 years of age Or continue just in DM or CAD You are not wrong to stop statins in patients > 75 years of age, in the absence of a clear indication. Is demand ischemia considered a true MI I have heard differing opinions, some based on the peak troponin level. Ischemia is not infarction, so demand ischemia is not a true myocardial infarction. For High risk patients who gets controlled cholesterol with treatment and their LDL goes < 70 do we stop the statin No. Their risk factors have not changed. for familial HLD (LDL >190) at what age can youdo you start statin therapy high intensity or moderate intensity the American Academy of Pediatrics recommends starting treating at age 8. Use high intensity statin. There are limited guidelines on triglycerides. It is not in the ASCVD score. The recommendation is to follow what medications used to manage hypertriglyceridemia the guidelines for LDL and total cholesterol. Can you explain how to interpret the troponin delta There are alot of people with elevated troponins that are not dt ischemia You have to interpret the results in the context of the entire clinical picture. BMS are used for "main pipeline" lesions, such as LAD; focal stenosis; or a patient who will not adhere to a minimum of six months of DAPT. what are the indications of regular stent vs drug eluted stent For everyone else, DES are used. Yes, the BMS will occlude sooner. Endotheliazation is complete within one month, so not much is gained what is meant by DES keeping it "open" longer does that mean that with BMS they close up quickly with DAPT beyond a month. For the DES, that process takes months so the counter effect of DAPT has to and close up meaning occlusion go on for months. Routine use of Aspirin, in high risk patients, but have no CAD as yet. What should you tell your Depends what the "high risk" is. If it is diabetes, or PVD, etc. then they have an indication for aspirin. You have to try to determine if it is new, using the clinical picture and troponins. If it truly is new, it should so new LBBB can be treated judiciously be treated as new acute coronary syndrome. if starting pt on statin when should we recheck a lipid panel No firm guidelines on this. Can check in 3-6 months to ensure adherence. Guidelines suggest starting screening at age 40. The only indication for screening at a younger age is history Can statins be started in pts younger than 40 yrs of familial hyperlipidemia, or in the development of atherosclerotic disease. Do you need to be prescribing CoQ10 with statin to help combat myopathy Science behind this, if This is a common question, but the evidence behind it is limited. Aren’t statins diabetogenic So why put someone on a statin who can get DM yes, statins increase the risk of diabetes, but the risk is only 0.39% over 4 years. NNT 255. For "main pipeline" lesions, such as LAD; focal stenosis; or a patient who will not adhere to a minimum of When would a bare metal stent be used six months of DAPT. There is a theory that if the beta receptors are blocked, it exposes only the alpha blockers to activation by Can you not give labetalol or atenolol in cocaine induced coronary problemshypertension cocainemetabolites, leading to a hypertensive crisis.

Challenging Issues in Hematology I am not aware of a change to criteria since the addition of the ADAMTS13 which aides in dx and is listed in isn't criteria for TTP changed the second TTP slide vaccines needed for MM Pneumococcal, HIB, flu Can you comment on the new sickle cell treatment with bone marrow transplant partial that have been successful so far. Done in conjunction with NIH I am not aware of this but will certainly research this- thank you With confirmed iron deficiency anemia in a young woman, can you assume menstrual blood loss or Retic count helpful here- if not having menstrual bleeding should see improved retic count in 5-7 days- if should rule out GI loss (fobt) too not, eval for ongoing bleeding what is work up for "bite cell" on a smear Need hemolytic anemia eval- retic, fibrinogen, LDH, coags How many RADs for the full-dose CTPA Great rundown of the # of CXRs, VQ scans and low-dose CTPA. Older scanners Really depends of the machine- most have about twice as much radiation as the newer low dose scanners Define etiologies of a nonsurgical related provoked VTE Estrogen use is most common, pregnancy, cancer how anticoagulant PE output DOAC or Coumadin with heparin bridge Chest guidelines recommend LMWH, low dose UH bid or tid, or fondaparinux for acutely ill hospitalized Can you discuss VTE prophylaxis in hospital setting patients with limited mobility If high index of suspicion for isolated pelvic DVT, then yes but usually pelvic DVT associated with proximal lower extremity DVT

US negative but high probability, elevated Well's, positive ss, do you then get a CT angio abdpelvis Otherwise, repeat venous Doppler in 5-7 days Can you repeat what you said for cancer associated thrombosis Why is Lovenox preferred 9% clot recurrence on lovenox vs 17% on Coumadin DOACs would the compression stocking help with edema of LE or not worth the cost in pt with dvt Stockings good for chronic edema from clot but not acutely Do we repeat a VQ scan after treating for PE Not recommended Is DVT prophylaxis indicated during an inpatient stay Depends on mobility status- if near normal mobility, not indicated what does "increased time in therapeutic window" as a reason why DOACs are preferred For Coumadin- even adherent patients are therapeutic (appropriate INR) only 60% of the time In hospital, should not order SCDs anymore if no compression stocking benefit Or keep bc No data for stockings if treating for VTE- still recommended if pregnant and anticoagulants stopped for intermittent squeezingrelease of TFPi delivery Compression is a no for treatment but what about prophylaxis that’s still yes Yes but not as effective as anticoagulants for preventing VTE what is CTPA Computed Tomography Pulmonary Angiogram- the cat scan to evaluate for PE How long should the immobilization be How many Hours of sitting Immobilized for 3 days or surgery in the last 4 weeks Surgery was 8 weeks ago so often DDimer normalized- it's a non invasive test with great NPV so still first this patient is sp lap chole isn't d-dimer going to be elevated anyway line in diagnosis For DVT dx Point of care ultrasound (US) vs official US Depends of expertise of operator for POC US- studies on formal venous compression studies Would we need to remember all components of wells criteria or will the stem be fairly obvious See above answer Will we need to know the specific criteria for WellsModified Wells Stem will be fairly obvious- low risk for DDimer or high risk for US for DVT and CTPA for PE Not if strongly associated provoking dx like on estrogen, post CVA, immobilization, etc. If lower risk For a provoked DVT prior to age 50 do you need to work-up for thrombophilia provoking factor like a long flight, I would test If DVT US unable to visualize vein due to soft tissue swelling what is next test Repeat in 5-7 days, empiric tx if high pre-probability wells score

Commone ENT Problems Ear perforation AOM can oral abx cover both What if they have strep and TM perf Best tx Amoxicillin covers both Why is augmentin not first-line for bacterial rhinosinusitis It is according to IDSA- AAFP says Amox or augmentin first line What fluoroquinolones to rx sinusitis Levaquin preferred does stroke show wrinkles on the forehead Yes- unless you have a bilateral CVA Check out you-tube for a great video- we used to show it during this lecture but with all the new content it What is the Dix-Hallpike Maneuver had to be cut out to fit into 30 minutes -( What is considered recurrent AOM See below For tube referral, is the recommendation 3 AOM infections in six months in less than 4 years of age 3 episodes in 6 months or 4 in last year with at least 1 in the last 6 months if 10 yo has a cough keeping them at night with dx of flu or strep throat, would you treat Strep with cough is pretty unusual so make sure dx is correct supportively or OTC cough medication Flu with cough- yes supportive tx with - no OTC what's the best way to dx acute sensorineural hearing loss in the office Hand held audiogram No data for acute can we use PO Prednisone or PO steroids for acute or chronic sinusitis Data for chronic only if co-existing nasal polyps I am not aware of guidelines on this- I go to daily Claritin and then decrease the decongestant part to every how do you wean patients from controlled substances like claritin -D other day for a week or 2, then twice a week for a couple weeks, then off the decongestant dix hallpike manuver is specific or sensitive Has best data on NPV for BPPV- if negative hallpike, it's not likely BPPV neurological findings in history and physical should we do Vestibular rehabilitation or rather Epley's maneuver. Vestibular rehab How long do you do complete voice rest for hoarseness Typical is 4 weeks- less for rapid responders who can continue home therapy

Common Neurological Disorders Yes- but studies vary 20-70% of adult patients with refractory epilepsy showed >50% reduction in SZ events but very low adherence to diet due to restriction- some studies show 82% stop. Risk of CV impact My son has been on the keto diet for two years for seizures. Have we considered this with adults given high fat diet IV reglan for refractory migraine Yes- listed on the second line treatment slide Yes- brain cancer better evaluated with contrast- I will make that clearer on that neuroimaging slide- thank I thought contrast was used if checking for a cancer you! new onset after 50, does that apply for tension and migraine No- just migraine Do I need to stop "triptan" treatment of migraines, post thromboembolic stroke Relative contraindication due to raises BP so mostly ischemic CVA Is "Chronic daily headache" the new term for rebound headaches Yes In assessing if a patient had a seizure vs. myoclonic jerks, is it possible to still be conscious if the shaking is bilateral Yes When is it a good time to stop AEDs See 3 questions above Should routine blood levels be done on seizure medications and how frequent once they are stable on them Yes for first generation AEDs- follow package insert recs How long is a patient at risk for non-convulsive status epilepticus after being transferred out of the Highest risk first week If a question asks "what's the next best step" between ordering imaging vs eeg for seizure, what is the correct answer Imaging- this can show urgent stuff like CVA bleeds that need different immediate treatment Nexplanon has lower serum levels of progesterone than depo provera so that is likely why there are higher you list IUD and depo provera as good contraception options. why not nexplanon pregnancy rates with AEDs with nexplanon than depo If EEG shows absence in a kid- use ethosuxamide or valproic acid what eeg findings would lead you to prescribe one seizure med versus another What about MRI If focal SZ- better tx with oxcarbazepine (<16), carbamazepine, keppra, phenytoin, zonegran (16-59 yo), findings gapapentin or lamictal if >60 SIADH cause seizures by dropping Na+ Yes if acute Na drop 2 years if SZ free in general- consider long term if high risk job like working on power lines or high risk SZ When can you wean off antiepiletics in general population if no new seizures types or abnl neuroimaging Can alcohol withdrawal wax and wane Usually theses waxingwaning Sx are due to the BNZs used for treatment wearing off Seizures with mences---why does this occur Unclear- menses associated migraines and pseudo-seizures much more common than actual SZ Better for dementia eval but rare for SZ- extreme hyperthyroidism is on the list of rare causes but other for sezures you dont check B12 or folate, TSH findings on history and exam should give hints to that CTA ordered by neurologist for stroke workup, do you still need Carotid US CVA talk is tomorrow Monoclonal abs Will that come out in boards Not likely- just know oligoclonal bands not required for the dx of MS What about ck for seizure diagnosis Could point to some types but many types have no impact on CK like atonic or non-convulsive eeg.... need sleep deprived Not initially- if no EEG and recurrent SZ, then consider sleep deprived, video EEG

Common Newborn Issues What are the recommendations concerning helmets in flat head in the newborn If positional placiocephaly persists with conservative treatment would send for evaluation. Do you need to follow Breastmilk jaundice with labs since it falls outside the nomogram How do For breastmilk jaundice in a health baby, just follow up until normalizes - usually at 1 month and 2 months you decide when treatment is needed of age. Breastfeeding jaundice - feed more. treatment with both breastfeed jaundice and breastmilk jaundice is to just feed more Breastmilk jaundice goes away on it's on, feed normally. Good question - with ranitidine under voluntary withdrawal, you can still use it but might want to switch to For GERD, which H2 blockers are you recommending now famotidine on cimetidine. what is the name of the music piece playing now Huh - I'm not hearing music.... What do we do if Ortho referral takes a long time to go through for CHD Should we order imaging in the meantime The ABFM answer is orthopedics referral. In real life you would still only do imaging if you have a question. what is the dose of miralax for babies Only use after the age of 6 months - dose in slides. Is it okay for an infant to sleep in the prone position after they can roll over Yes - still put them down on back. What's the most common sign of encephalopathy in a newborn lethargy, irritability poor feeding, seizures. Do you have a cut off for when to start IVF for tx for hyperbili (2 or 3 pt's above the phototherapy We don't start IVFs - just make sure they are feeding well. Any pearls for motherpatient education on the subject of Neonatal Abstinence Syndrome in our A whole talk - I would just make sure you have a protocol that you follow. If your hospital doesn't have one pain or Substance Use Disorder population it would behoove you to help institute. lacrimal duct effect conjunctivitis not usually at problem. It's a blocked tear duct that collects tears so eye is more watery. What is the value of having baby sleep in a baby co-sleeper next to mom Ease of breastfeeding. Is breastfeeding jaundice the same as physiologic jaundice Breastfeeding is an Exaggeration of physiologic jaundice - caused by insufficient feeding. Preterm infants - up to 1 year of age. How long do you continue iron supplements in infants Term breastfed infants - until 1 year of age or eating enough iron rich foods Usually infants with bilirubin levels of greater than 10 will have a yellow-orange cast to their skin, but you how is the color of stool correlated with jaundice newborn with elevated bili need yellow coloring can't rely on that, which is why universal screening is recommended. what dietary recommendations do you give to mom when infant has GERD Try eliminating allergens - eggs, wheat, etc. History, physical. If meconium at delivery, consider meconium aspiration. If mother was ruptured great how do you rule out of other causes before dx with TTN than 18 hours, had a fever, etc, think infection. It's a consistency issue not a timing issue. If the stools are small and hard, that is constipation. If it is just How do we define constipation in infants After how long do we say they are not stooling enough infrequent, but stool is normal, not constipation. How long to give vit D for formula fed babies. and mom who breast feed should stop vit D once they start vit D fortified on same day Need to be drinking 32 oz or 1 Liter of formula daily, typically this is at around 4 months of age. Mother has not been DAIGNOSED with herpes. Many women may have it and just don't remember having how do babies get Herpes if mother has never have herpes an outbreak.. Mother wants to use breast milk but is not producing enough; any recommendations on obtaining This is done often in the NICU - using donor milk. Mostly if mother is not producing enough she needs to other mothers excess breast milk Safety feed more often. Breast milk is all supply and demand. Also make sure she is drinking enough fluids.

Emergency Medicine I Septic shock can used NS or lactated ringer Either At my hospital, only orthopedic surgery canwill measure compartment pressure. Any suggested alternative ways to test Depending on who’s on-call, they don’t always come in overnight. There are a number of easily available disposable products What if antibiotics are available before cultures have been drawn Do I start antibiotics despite no cultures If so, how long do I have to get cultures after starting antibiotics Ideally get your cultures ASAP, then start Abx Do you see greater tissue swelling when irrigating with water versus normal saline No cost & availability limit use prothrombin complex concentrate $4000-5000 How can I apply 30-40 mmHg compression stockings without causing compartment syndrome This force may be applied to shin but is not the pressure within the compartment HOW TO TREAT SEPTIC SHOCK IN AN ESRD PT ON HD IVF OR NOT If you document patient has ESRD, you do not have to give 30ml kg how long would you tell pt to expect seeing improvement of subconjunctival hemorrhage and how long does it take for resolution Typically 10-14 days... like any bruising under the skin-it will change all different colors Do you give vaccines after or before splenectomy, as slide notes "14 days beforeafter" If elective splenectomy, either... if traumatic, you only have one choice Do hyphema and subconjuntival hemorrhage look the same on exam No...see my slide pictures at least 24 hours, but if you are using for life threatening hemorrhage, doubt you are going to want to start How long does that PCC then take to wear off if we need to anticoagulate the patient again. anticoagulation that soon since epidural hematoma pt has normal behavior at first, then downhill fast would anyone with head injury needs ct head No.... they will often be deteriorating in front of you. Base your decision on CT on clinical findings

Emergency Medicine II Do you use txa for angioedema No data I am aware of why glucogon if on a b blocker when treating anaphylaxis with epi Theoretically need to reverse b blocker because epi will not produce b-agonist effect you want The study by Sinert R j Allergy Clin Immunol Pract 2017 shown in a study of 121 patients that I Catina this How about bradykinin inhibitor as treatment for ACEI induced angioedema provided no benefit How do you diagnose cigutera poising Symptoms only or lab test Treatment Clinical... there is no lab test How do you distinguish scromboid poisoning from anaphylaxis Scombroid will only give some skin changes... this is different than anaphylaxis do we need to know dosing for epi for adults and childern I don’t know... simply 0.3ml IM - that’s what is in the EpiPen how long will you observe the pt in ED for possible second wave of anaphylactic reaction No right answer Can we go back to the ACE after an episode of angioedema. No..no..no is cat scratch disease different from cat bite treatment because of no open wound No.. completely different organisms! Do you give FFP in angioedema There are only a few case reports supporting this Is genetic resistance to mrsa-ca possible, with no formation of abscesses Not that I’m aware of

Fever and Infectious Diseases in Children When can the kid go back to school child care after hand, Foot , mouth dz Once fever free for 24 hours without NSAIDSacetaminophen - and they feel well enough to return. At what age can a child request a vaccine against parental consent 18 years - legal adult. They often come in before heading to collegemilitary to get caught up. Does it need to be Buckwheat type of honey see answer above I would guess so - but it comes in a lot of different strengths and liquids. We keep some liquid in our office Are Dexamethasone tablets as effective as liquid if the liquid isn't available (crushed up) which is meant for injection but in a pinch we have given it orally as well. what is "buckwheat" honey see above answer to same question How would you work up recurrentintermittent lymphadenopathy with no other symptoms If it persists longer than 4 weeks without obvious cause - would start the work up. If the coronary aneurysms persist, this can continue into adulthood. As to higher risk of developing atherosclerotic heart disease - jury is still out, as the disease was only name 40 years ago, which means Does kids that have Kawasaki are they more incline to have adult vasculopathy those patients are just now hitting the age they would develop ASCVD. How do you treat post-strep reactive arthritis in children NSAIDsacetaminophen Some liquid Dexamethasone preparations have a small percent of alcohol (2-5%), is that still safe for infants and kids I would make sure you can find an alcohol-free version in your office. if in an adult with centors score of 2-3 , rapid strep negative, do we still need do strep culture Nope - only in kids. Do we need to give antibiotics to contacts of meningitis (like dorm mate) Yes - rifampin , ceftriaxone, ciprofloxacin for close contacts within 24 hours of diagnosis. You can use in kids but remember that you need to culture for a negative rapid strept so that is a Centor criteria meant mostly for adults. Pediatrician recommend not using. What’s you opinion on difference. Plus, based on the age of 3-14 gives you a point already so just one more symptom gives you a that score of 2 which means testculture. A single dose of oral dexamethasone (or IM) can decrease pain in the first 24 hours for kids over the age of What's your opinion about using dexamethasone 10mg IM for symptomatic relief for pharyngitis 5. Would only use for a significant sore throat so that benefits outweigh risk. I had a question about using apple juice for oral rehydration (ORS). Is it correct to use if no access to Pedialyte or Gatorade Yes - can mix apple juice with water (half and half) The new PNA recs is to tx with Amoxicillin and no longer Azithro for CAP. What will be on the The new guidelines are focused on adults. I'll look again but haven't seen anything new on kids yet. Likely boards will come soon. Peds do not use procalitonin like adults. Do you recommend We actually tend to use it more with kids than adults as a marker for serious bacterial illness. Any questons potentially about "partial Kawaski" and management inpatient vs. discharge from ED Unlikely - usually the boards will give you a straightforward case. Is there harm in using a bronchodilator for a child with bronchiolitis There are side effects (tachycardia, etc) which is why it is no longer recommended. was she the BEST resident you ever had Of course) That is what the studies were on, likely due to funding. Practically I recommend any honey but the boards What is special about buckwheat honey compared to other honey will specify. If it is a simple febrile seizure (less than 15 minutes, associated with fever, generalize tonic clonic seizure) What are the recommendations for admission in a febrile seizure pediatric patient no admission needed unless they have an infection that necessitates admission. Did she always talk this fast Always - ask my kids or residents! Or husband. Isn't there a Baby Vicks Yup - it is essentially a non-medicated ointment with some herbal additions. uti first line is bactrim or cephlo Either is fine. In the ITE 2018 a q about HSP, a patient had some grade of proteinuria, but the answer was not A lot of the information is from adult studies. You don't give steroids for all kids with HSP to PREVENT treatment with prednisone nephritis just those that develop kidney involvement. So we call as UTI is culture is more than 50,000 in kids Yes, see above answer how do you obtain a urine dipstick in an infant 21-90 days Do you cath or just use non-sterile Cath urine is needed. You can't do a "bag urine" what happened to over 100,000CFU ml now 50,000 Yes that is from the AAP 2011 guidelines why procalcitonin It is a systemic infectionsepsis marker.

Heart Failure If a patient is on entresto, by how much does that affect BNP levels Varies with individual and clinical picture. The meaningful outcomes are death and hospitalization. Why not Ivabradine instead of Entresto recommendations are based on the trials. I'm confused over the statement that furosemide may reduce mortality Are there any studies that There is some evidence it may reduce mortality. does Lasix reduces mortality There is some evidence it may reduce mortality. do we add ivabradine to beta blocker or replace the beta blocker It is added. Should you maximize ace or arb dose before starting anri Yes, as tolerated. If in Afib with RVR in heart failure concerning for exacerbation, l, what is the first line treatment in this case for rate control usually metoprolol. in a pt with normotensiveslightly HTN w HFrEF, diabetes, is it better to maximize acei dose wo BB or lesser doses of both They should be on BOTH of these medications. In a patient with no other risk factors,african american on lisinopril induced angioedema and hctz, If the patient has HFrEF, they need to be on an ACEi or ARB. So they should be switched to the ARB, rather is it safe to use losartan, or would adding a calcium channel blocker be appropriate than the CCB. do you treat HFpEF the same as HFrEF No. We don't know how to treat HFpEF. In patients with diastolic heart failure, do you recommend same drug treatment as systolic heart No. We don't know how to treat HFpEF. Might measure when the patient is stable to establish baseline, but symptom control is more important Do you monitor Bnp to see if heart failure is improving and if so, how often than following the number. The main concern with NSAIDs is the effect on the kidney and fluid retention. Steroids do not have that can you use prednisone in HF patients given cannot use NSAIDs. effect so can be carefully used. Challenge in clinic coverage for Medicare for BMP - a lot of ABNs - any coding help for me Odds are they have HFpEF, since so many people do, so you could use that in coding. Sometimes questions will say "patient has class III heart failure.." Assume "class" means NYHA On the exam, they will specify "NYHA" classification. They will use the term. What are the current recommendations for treating a patient in the EF gray area of EF 40-50 Will this be tested There are no firm recommendations in this "gray zone," so it is unlikely there will be questions on this. Is not EF of <30% indication of ENTRESTO Threshold for treatment is 40% (HFrEF) in the presence of ongoing symptoms. Yes, no need for the "wash-out" that is indicated for Entresto. It is the sacubitril part of Entresto that if angioedema with ACEI, can switch to ARB right away increases bradykinin, not the valsartan (ARB) part. What does "negative inotropic effects" exactly mean It means it makes the pumping less effective. frustrating since it is integral in diagnosis. What do you link too if they do not have underlying hypertension or known CHF Odds are they have HFpEF, since so many people do, so you could use that in coding. You can hold the beta blocker if the patient is unstable, especially if the patient is hemodynamically Should we continue b-blockers in pt with CHF exacerbation and sings of fluids overload unstable, but as soon as it is stable it should be restarted. Will any dose of beta blocker or ace inhibitors beneficial for CHF or is there a particular dose necessary to confer benefit Any dose is effective. The dose is titrated based on patient tolerance. Is gynecomastia with spironolactone dose dependent Yes How do NSAIDs cause a heart failure exacerbation Fluid retention (effect on renal function). Would you start spironolactone over furosemide due to confirmed mortality benefit Yes Should we memorize names of specific research trials for exam No What about pt with diabetes On insulin Can you still use Beta blockers Which ones Yes, you can use the second generation beta blockers. I was told not to use Lasix in HFpEF because these patients are preload dependent and Lasix will decrease the preload. Is this a true statement Yes. do you prescribe ACEi for patients with HF in absence of HTN Yes! It improves outcomes, regardless as to whether the patient has hypertension. If the EF is > 50% and they have evidence of thickened wall and impaired filling on ECHO, they have What degree of diastolic dysfunction is HFpEF diastolic dysfunction. WHY IS LOSARTAN GETTING OUT OF MARKET Due to manufacturing concerns (contamination). What is the terminology for patients with EF between 40-49% Borderline EF, or improved EF. They are both Class I LOE A recommendations. Generally, the spironolactone is used first, but clinical Is bidil recommended before spironolactone in AA judgment should prevail. Any need to know difference between BNP and proBNP No. Which beta-blocker is best in DMII to avoid masked hypoglycemia Use the second generation beta blockers. Is following serial BNP levels a sign of heart failure recovery The trends matter, but the main outcome to monitor is symptoms. It is also helpful to establish the Ie a bnp of 3000 vs 2500 or getting worse at 4500 patient's baseline. What is the role and importance of utilizing BNP vs pro NT BNP in HF BNP and pro-BNP are not interchangeable. recommendations still evolving, and it is important to be And how about HS-CRP vs regular CRP in ACSCAD consistent in which measurement is used.

Hypertension I know it is not the guideline but if we do get a baseline Cholesterol profile on a 28 year old and find that after proper fast, the TG's are 400. Management> There are no guidelines on triglycerides. for exam purposes what is the best test for renal artery stenosis There is no preference. What about study that showed “white coat HTN” still should be treated bc likely have HTN at other It is good to think about, and a good reason to get more data. You can look at other risk factors andor times when anxious untreated is more dangerous patient preference to help you in the decision as well. What meds for renal artery stenosis. Same as for essential hypertension, or most other causes of secondary hypertension. Why was sleep study the wrong answer on the first question, instead of reninaldo ratio The patient had a BMI of 24, so not really the body habitus for OSA. Another clue was her hypokalemia. Any increased risk of hypokalemiaother electrolyte abnormalities with Chlorthalidone vs. HCTZ Chlorthalidone is more likely to cause electrolyte abnormalities because it is more potent. When treating HTN, should you max out one drug before adding a second and so on Yes, in general that is true, but it is more effective to add a second drug. How about ace I or arbs in ESRD patients Yes, they are indicated. You'll hear more in the renal talks. occasionally but causes hospitalization. Why does the jnc guidelines not recognize this in patients Yes, that is a common problem, but the other medications cause problems, too, and there is good data that over 70 older adults benefit from diuretics. In older adults, it is important to monitor electrolytes. African american with microalbimunuria first line treatment for HTN is CCB thiazide or shoulde we Since there is the indication of microalbuminuria, the patient should be on an ACEi or ARB. That indication give lisinopril for its benefits with microalbuminuria overrides the other guidelines. If patient clearly has WC HTN, should it be coded for since their in-office elevated BPs will likely quiet sitting, in order to minimize this problem, and to give the patient a chance to register a normal cause dings reading. What is the best treatment treatment for Diastolic HTN Same as for systolic HTN. Regardless of what guidelines we use in practice, will the board exam go by JNC8 guidelines or AHAACC guidelines JNC 8 guidelines. would you order renal doppler duplex ultrasound or just renal ultrasound either is acceptable. There are no specific guidelines on this. How do you obtain ambulatory BP monitoring None of the hospitals or cardiology groups in our and Medicare will not pay for it. So, if you don't have a device to loan out, have them do home blood area provide this service. pressure monitoring.

Managing Dysrhythmias says no AV nodal blocking agents for aflutterafib. Is this just in case you can't recognize WPW on Avoid AV nodal blockers if the tachyarrhythmia is aflutterfib in a patient with a delta wave. If they do not EKG have a delta wave, they will not develop WPW so can be treated with nodal blockade for afibflutter. Is there a possibility that LQL syndrome could show up on the exam Do you mean LQT Yes, that could be on the exam. A QTc greater than 500 ms has a twofold to threefold increased risk and each 10-ms increase contributes What is the likelihood of a prologed QT changing to torsades toe another 5 to 7% exponential increase in risk. It probably happens more than is realized. vtach cannot use CCB ike dilt on the test CCB is not a treatment for Vtach - use lidocaine or amiodarone. is MAT a type of SVT Technically it is, but by convention it is not what we mean when we refer to SVT. Transient complete heart block after RCA PCI, will you consider pacemaker Not necessarily, as there could be recovery. the most effective beta blocker for rate control is metoprolol (either succinate or tartrate), so that is Which beta blocker has least effect on blood pressure if chosen to control hear rate in SVTs usually what should be used. The dose can be lowered if patient develops symptomatic hypotension. Do you need to screen patients whose 1st degree relative has WPW Yes. Although there is no general agreement in what to do if that relative has delta waves. Good to know. Most people will choose that rather than flecainide, propafenone, dofetilide, or sotalol which "may be reasonable" per the guidelines. But as ablation therapies continue to develop, this is Recent WPW question on 2019 ITE answer for best treatment was catheter ablation understandably evoling into the preferred treatment. how does the morphology of a delta wave and q wave differ a delta wave is that front notch of the QRS complex. So the delta wave is part of the QRS complex. If the left atrium is large, the likelihood of success is lower, so it may not be a justified procedure. Patient what are the contraindications for catheter ablation in Afib preference is important, too. Depends on the rest of the clinical picture. If it is a healthy person with no suggestion of an MI, they don't So does anyone with Mobitz Type I needs workup for inferior MI If so, cath or stress test need work-up. SVT is usually much higher rates that sinus tachycardia. Also, sinus tachycardia always has a cause, while SVT vs Sinus tach. How can you tell the difference SVT can just happen out of the blue. Has not been evaluated in an unbiased manner (the studies are sponsored by Apple). There are anecdotes, Do you trust apple watch 5 to detect or monitor Afib but studies don't have a control arm and don't access affects of false negatives or false positives. Depends. Sometimes, when it is not tolerated, it is just stopped. But if another medication is going to be Are they any guidelines to wean patients off amiodarone used, the transition has to occur in the hospital so the rhythm can be monitored. Many of them are already on anticoagulation because they have had the condition for a while (ablation is not an emergent treatment, rather is done is patients who are not satisfied or responding to other treatments). Ablation is also not like cardioversion, it doesn't always act immediately, so it is not as likely to Do you need to anticoagulate before and after catheter ablation dislodge a clot as with cardioversion, Still, most of these patient are on anticoagulation already anyway. Good question! The end effect would be the same. Maybe just because it is in the CHA2DS2-VASc score Why not just take out the sex component from the cha2ds-vasc equation ... and they don't want it to be CHA2DS2-VA as that sounds weird. So patients who are in community and nursing home who are on warfarin now should be switched Use your clinical judgment and consider factors such as cost and convenience. The frequent INR checks are to DOAC easier in the SNF than with ambulatory patients. Certainly can adjust based on patient and provider preferences, but the official recommendation is to use It seems expense & availability of reversal agents would weigh into decision of DOAC vs warfarin the DOACs. how do we change Afib pt from warfarin to DOAC bridging dosage stop the warfarin and start the DOAC as soon as the INR is less than 2. guide treatment. Generally, warfarin has been used, but DOACs can be used, just depends on the kind of cancer, what the treatment is, etc. Several studies are looking at DOACs vs heparin for DVTs in cancer anticoagulation in cancer patients--- Warfarin, LMWH or DOACs patients. If Afib is rate controlled but still in afib will that still cause atrial remodeling after weeks to months Yes. Yes, with close monitoring. Eliquis is anticoagulation and plavix is anti-platelet and the indications are Can you use Eliquis and Plavix together somewhat different. Patient may have indication for both. If it is large, they likely have atrial remodeling and the ablation is not going to hold. They have likely had the Why is Left atrium best predictor of long term success in a-fib condition too long. What is the goal rate for rate control for atrial fibrillation Less than 110. How many times do you try adenosine before changing to other medications to treat SVT Generally it is give twice. You would do pill in the pocket over catheter ablation Are cardiologists still recommending this You would use it based on frequency of the SVT and patient preference. How can you distinguish between Vtach and SVT with aberrancy Important to make distinction (ie. It is not a problem to try adenosine first. If they become hemodynamically unstable, they have to be Treating VT with adenosine not cool!) electrically cardioverted. You are right, it should not be used. It is not a first line choice anyway. The other person who teaches the Why are negative chronotropics NOT a contradiction to bradycardia cardiology part of this course likes the question for the cyanide joke. I can't get him to take it out.

Well-Child Care and Adolescent Issues what is considered anemic range in child See answer to same question above What is the best position for baby to sleep on back So is it ok to have HIV, Syphilis testing in a sexually active 13 yo male (3 girlfriends!). I reckon him high risk Absolutely! Would make sure you know a counselor in the area to refer. We have a lot in our practice and we start how do we deal with gender identity in our teens there was ITE question this year about this with counseling before we do any medical interventions. In sports physicals for makes, we do hernia exam for male athletes, but what if we notice an abnormality on the testicle. I know we can't address at that specific that visit, but isn't that kind of 'indirect' screening in a sense If you are noticing an abnormality you are no longer screening so you would work it up. What is the dose of fluoride supplementation How do you dose it Remember this being on the intraining exam always. There is a table in your handout - it is based on age and how much fluoride is in the water. What are good ways to screen for bulemia Ask about body image Has there been mention about disordered eating It has been added to dsm5 I haven't seen yet on boards but will keep checking. checking tanner stages until what age until they are stage 5 (adult development) what is the name of the app for BP for kids Ped BP - my personal recommendation, not necessarily from the AAFP what is the connection between necessity of contraception and Ace inhibitors in teen females see answer to above question This might be discussed later, but we aren't to check HPV if less than 30 yo, but the ASCCP guidelines for 25-30 yo is guided by HPV results. How is this reconsiled There will be a talk on this. What is the correlation with ACE inhibitors and contraception in females See answer to same question above Do we have to memorize all developments milestones I would definitely look them over. Speech milestones seem to be the most common. how long should we wait if a child was noted to have some developmental delay before referral Don't wait - if you have a concern refer immediately. what did you say about measles and seizure The MMR vaccine has a slightly higher risk of febrile seizures. Ace inhibitors are not considered safe in pregnancy - and this age group is at high risk unless using effective why no acei in young females contraception What about ASCUS before 30, do we screen for HPV then There will be a talk on this. when do we get ECHO is kids who play sports and noted new murmur on exam I believe this will be discussed at the MSK talk. HIV and syphilis - blood draw in adolescents Yes Is parental consent required for abortions depends on the state. what vaccines do kids usually age out of I would recommend the vaccine table which lists the range of ages for all vaccines. How much correlation have you seen in obesity and history of abuse Not so much. Ok to have some hand clenching just not the majority of the time persistent fist clenching is concerning. Assuming this is a term 6 month old and not a premature and calculated gestational age is not 4 months, correct Not sure what you are referring to what labs do we obtain (should we even obtain labs) if you have a child with BMI greater than 95% Can consider A!C, lipids. Should you avoid fluoride (bottle water) or use fluoridated water for bottle making use water with flouride would we be asked how teeth develop, which teeth erupt timeline not that I have ever seen what is "instrument based" Red reflex, coveruncover test - basically you looking at their eyes what levels do you consider anemic and empirically treat They are different based on age. Follow the normals by your lab or there are tables online. Do you feel using jumpers in babies can lead to hip abnormalities Do you recommend using them I have not heard anything specifically about that. What about use of imipramine TCA for bed wetting What does evidence say about that It can be used but would be an option after alarms. No specific evidence - would basically look for Tanner staging. I explains to teens that I'm just checking Resident here - differing attending opinions. At what age do you do or not do genital exams their development and let them know if they will like grow more, start their periods, etc. if the child is 15 yo do they get 2 or 3 doses of HPV If they start at 15 or older - 3. How would you address a mother if she wants to check her daughters virginity That's not a thing - there is no way to check reliably. I still clench my hand. Should I be concerned Only if punching someone screen time with kids who use computers for school work Think about the limits for optional screen time. What are your thoughts about when to allow a child to have their own internet connected mobile device. Is there any evidence about age to allow Up to the parent and their assessment if kid is ready. Need to counsel on internet safety. How do you realistically screen a 3 year old's vision in the primary care setting You can try to get them to identify pictures. It's tough! Some kids can do it no problem, some can't. Do you need to perform MCHAT twice or just once within that time period Once at 18 months and once at 24 months. If you do AAP recommended universal lipid screening, under which diagnosis do you code often not covered where I work unless kid is morbidly obese &or FHx Since USPSTF makes it an I recommendation, you may not get it covered. When do we stop doing red reflex testing Usually 4 months, once you do coveruncover test.

STIs, Vaginitis, and Vaginosis Finding signs of vaginal infection (discharge, odor) during WWE, do you obtain a Pap Smear or treat and do Pap Smear later Do the Pap test that day. If a patient has genital HSV for the 1st time, should they be on prophylactic acyclovir Not recommended. Treat first clinical episode frequency of screening for Hep C in person born in 1945-65, after initial negative test, and NO additional risks Its a ONE TIME Screen Why not retest men like women for trich in 3 months Data are insufficient to support retesting men (CDC 2015) Should patient continue to have sexual intercourse while being treated for any of these infections No sex for 7 days after COMPLETING therapy. How frequently canshould you screen pts with high risk sexual behavior for HIV I see multiple women who are sex workers. Screen for STI every 3-6 months. (CDC) Is HSV only transmitted via sex, also can it be transmitted to partner if they have no active Possible secondary to subclinical viral shedding. PCR PCR has a sensitivity of 93.2% and a specificity of 100%, compared with respective values of 84.1% and best lab test for HSV diagnosis PCR 100% for culture. would you treat partner if pt dx with HSV No

Acute CVA & TIA Your decision to work up syncope is based on patients risk factors and history, you are not working up the How to determine if that patient who had syncope is one of the 13 %, shouldn't we do the work up convulsion Pathophysiology.. TIA is an ischemic event... TGA often associated with emotional stress, but the What's the difference between TIA and transient global amnesia pathophysiology is uncertain has convulsive syncope been on the exam and if so no work up No work up.. have no idea what is on test, but is a common real world case If pt. comes to ED to get 'checked out' following an episode of focal neurological deficits but has since returned to baseline, what is the correct answer for initial imaging ct wo or MRI MRI ... even though CT is much more readily available What about early brain MRI to decide on thrombolytic MRI will not help in this scenario- and it takes too long for BP control in acute CVA goal of decreasing BP 15% WITHIN 1 hr or 24 hrs There is no time definition, typically rapidly If full dose ASA is given on the way to the ED and they are candidate for tPA can they still get tPA even though ASA was given Yes Is there still any role for Heparin in acute stroke sec to afib No No- see summary slide of minor strokeTIA - DAPT for 21 days, then monotherapy indefinitely, all other is there a recommendation for asa 81 mg and Plavix 75 mg for 45 days after acute ischemic stroke ischemic stroke get Monotherapy indefinitely If a patient is on ASA previously and is now 60 yo or older do you stop it This best handled with shared decision making with your patient ThoughtsEvidence on TIA called a "mini stroke" Can't comment because there is no such thing in our world as a "mini stroke". Either you had a stroke or for after CVA diagnosis do we use ASA 81 mg ot ASA 325 mg daily Yes- either is acceptable Do you have to have MRI to dx TIA If available, it's recommended within 24 hts Carotid stenosis CTA vs Carotid US or both Typically you will start with US -- if positive, you move along would pt who are on eliquis still get tPA No ASA off as if somehow that pt has stroke shortly or sometimes after, it will be hard to face the pt. What is your thought Shared decision making with your patient Why are NOAC not approved for valvular a fib We're not studied at the time of approval.. Women w score of 1 CHA2DS2-VASc, Rx or no Rx Dysrhythmia lecture yesterday 0 for men & 1 for Women at 1 have option for AsA or DOAC, 2 or more DOAC women = no Rx; 1 men, 2 women = shared decision-making; 2 men, 3 women = anticoagulate. Men at 0 have option for ASA, 1 or more DOAC If a pt between 50-59 yrs old is already on a statin for many years before we meet them how do we know what their ASCVD risk score even is to make decisions about ASA Do we stop statin to get a baseline Or err on side of starting ASA You can recalculate CV risk score on statin Patients who are stable on coumadin do we have to change to a DOAC No... unless your patient wants

Adult Pulmonary Disease What do you typically give to cover for MRSA Pneumonia, and what you typically give to cover for Pseudomonas aeruginosa Pneumonia Local guidelines. Vancomycin for MRSA. Many choices for pseud. 65+ who gets Prevnar13 Above Do I need to test for TB for pre-employment physical exam in a patient who has been treated for No Patients w treated LTBI in 20s — and now back after subsequent years in high risk area. ASX. Needs clearance... Probably a good time to ask Pulm or Health department. can you please give the reference for no need for annual cxray with past tx of TB unless the pt is symptomatic as company will be questioning about it. thanks. CDC.gov or local health department. Is it appropriate to perform a CXR 4 - 6 week sp CAP treatment to evaluate for possibly missed lung No longer ROUTINE. You can order it if specific patient or risk. Can you comment on falsely positive quantiferon gold test secondary to frequent TST tests for health care workers. Not aware of that phenomenon, I am sorry. If PPD test and IGRA test results are the same why CDC does not accept anymore PPD test for Immigration physical exam Igra more definite. Pt w Acquired traumatic Asplenia <65years. would you use PPSV23 now and at 65 or use different dosing schedule Prevarication 13, then 23. What pneumonia severity score equals admission It is a risk score to guide you. >90 could be used in addition to your judgement. PCN allergic outpatient CAP tx does guideline say doxy, or as first line Either Do all patients with clinical findings of pneumonia in the office require a CXR for diagnosis, especially without comorbidities On the boards this would be recommended. so for CAP in adults we use amox or doxy and third line Can use azithromycin first if in low resistance area. The patient came from a country with TB and tested positive for Quantiferon Gold test but is completely asymptomatic. Does the patient needs treatment Get CXR. If negative, they need 6-9 months inch plus rifampin. pneumococcal 13 with patient >65 suggests joint decision making on CDC web site for those without risks. What does that mean Good question! Optional. But.... what about coronavirus! New for all of us i DO HAVE BCG VACCINE, THAT MEANS WILL HAVE PPD TEST POSTIVE. dO I HAVE TO BE TREATED Get Igra test. for our exam, pt being admitted with CAP you do not recommend procal. Are you going to discontinue antibiotics if procal is low Limited usefulness due to low sensitivity test. Could you repeat who more than 65 yo should get a PCV13 “High risk” patients.immunocompromised. CSF leak. Cochlear implant. Sickle. Malignancy. What is the definition of HAP Acquired within 48 hours of admission. Do we repeat CXR for bilateral or multi focal stable PNA treated as outpatient Clinical judgement. The guidelines are discouraging routine cxr. Would you consider a group home to be a chronic care institution and require Pneunovax That is a tough one. Not specific in the guidelines. so do we not give prednisone x5days for pna Consider in icu patients What if hyponatremic Test for Legionella Ag Test got Legionella if CAP is severe, or if needed in your clinical judgement. Just not routine in all patients. How do you use US in pneumonia Experts can use it to find consolidation or effusion. Do u dd Vanco if influenza is + Not unless other MRSA risk factors. For Tx of CAP and the treatment with Amox 1gm TID, how long is the treatment for Guidelines say at least 5 days, 7 days is typical. No exact recommendation. in the pediatrics lecture, we reviewed that grown childrenteens get azithro, should that change too Age up to 18 get azithromycin, adults you would follow the new guidelines. Do you think coronavirus will come out in the exam knowing this is very new We do not know. No specific treatment known so far.

Asthma: Pediatric and Adult To clarify, for acute exercise-induced bronchoconstriction, never use LABAs, even with inhaled steroids Would you clarify this vs pretreatment Do not use LABA ALONE. Can use combo therapy in pregnancy asthma slide. do these "increased risks" increase likelihood of asthma exacerbation or does asthma increase these listed issues Asthma increases those fetal issues What to use for infrequent exercise induced asthma exacerbation (couple times per year) SABA vs ICS-fomoterol ICS formoterol. SABA with a puff of ICS is an alternative. How is symbicort dosed as a rescue inhaler Twice daily how often should we use the ICS-formoterol in acute bronchospasm Twice daily for exam purposes is spirometry the answer or methacholine challenge test for asthma diagnosis or is trial with BD Spirometer. Challenge test is for when spirometer is not definitive Is Formoterol with ICS safe in pregnancy or just ICS Both category c how do you distinguish of dx for asthmacopd overlap Symptoms the question with the 14 YO with asthma symptoms, why is answer montelukast and not ICS+LABA Correction, ICS laba is correct What about recent AAFP article that Zpak can reduce inflammation in asthma exacerbations For people with frequent exacerbation. Recent study in CHEST and Lancet. Daily azithromycin for 48 weeks. Asthma exacerbation severe still IV mag Once. Which ICS is best to match with formoterol Any. GINA guidelines could it be applied to pediatric patients and starting what age Pens guidelines not yet updated, probably soon. Please shed some light on the " Risk of sudden death" with the use of LABAICS. More related with overuse of LABA alone. if pt comes to clinic in our exam, we would not use albuterol and use ICS Use duonebs for acute exacerbation What is the brand name of drug ICS-formoterol prn that Gina 2019 recommends for acute asthma management Symbicort How common do you see asthma present in adulthood Fairly often. Not sure why we are seeing more. So do guidelines recommend a combined ICSBeta for acute exacerbation or continue with prn albuterol + ICS daily Either, with oral steroids for 5 days Can you use Symbicort for rescue inhaler if someone is already on a scheduled daily ICS Controller inhaler (like Flovent) Interestingly, that is what the guideline says. Of the steroid inhalers, is there one with least side effect of voice hoarseness Not known. how often can you use symbicort PRN for asthma Twice daily Does Symbicort PRN use for asthma exacerbation apply only to adults or to both adultskids Adults at present How often can you use the rescue inhaler of ics- formotolol Q4hrs Twice daily So will the new changes be the correct answer on exam - example -Not Albuterol but ICS- Always use the latest guidelines so what is the rec for rescue in asthma ICS formoterol or SABA with an ICS simultaneous. Do you see spirometry changes in cough-variant asthma Certainly could. I thought reason for US not following GINA was studied dose of symbicort not the same as US dosing, is this true Under review by US currently Everyone is confused on new asthma tx guidelines. What will we be tested on Read GINA executive summary online for details. They emphasize all asthma patients need ICS. ICS+LABA are way more expensive than SABA alone, how to address this in clinical practice Difficult issue. Do the best you can in the patients situation Can you use albuterol (Beta agonist) in patient with hypertension or on beta blocker With close monitoring. Heart receptors are different type than lung beta receptors. So you can use ICS-formoterol for symptom control as well as a controller Yes Should hospitalized patients receive prn Symbicort now instead of albuterol nebs Hospital patients can still get nebs plus oral or iv steroids. for GINA recommendations, is there a max dose of ICS per day Thinking about kids on q2h nebs... There are once daily and twice daily ICS drugs. our exam will follow GINA 2019 Always use the latest So does this mean we should not be giving Pro Air or Ventolin rescue inhalers for our mild intermittent asthma patients for rescue anymore ICS formoterol would be better, works better, lasts longer. For the exam for acute bronchospasm should we go with Albuterol alone or new GINA guidelines Always use the latest guidelines. US is part of global GINA. Please review pneumonia vaccine recs once again. See the slide with the chart for best summary. Or cdc.gov

Behavioral Medicine I Some studies currently indicate 10mg celexa can delay dementia. Is that your go-to for depression in the elderly No, I use all the modern antidepressants what type of sodium rx needed if you think that is from SSRI. Will it need to be on if pt continues to Don’t regularly monitor sodium, because it is fairly unusual. If decreased sodium is detected, would switch be on SSRI how often do we need to check the sodium level to a different class of antidepressant which SSRI to use in breastfeeding and which SSRI to use in pregnancy All have been used, recent studies suggest sertraline, citalopram or escitalopram are safe What about ECT in the elderly How well do they respond &or would it be more beneficial if it means they don’t have to have side effectsrisks of Rx Very effective and safe in the elderly. Which SSRI s is better for elderly Can use any of them, no preference Is pseudodementia only in elderly Still do workup or try antidepressant first Do both. Patient may also have mild neurocognitive disorder How can you rule out pseudodementia Do the patient's usually pass a minicog or MOCA Usually only make the diagnosis in retrospect, after the cognition improves after treatment of depression Is hyponatremia with SSRIs dose dependent Not necessarily For Esketamine, if the pt gets high blood pressure do you treat it and continue the spray or just stop the spray If severe, stop the treatment. If mild and transient, can continue Is Esketamine twice weekly once, or every week for the duration of treatment Twice a week for 4 weeks, then decrease to weekly Thoughts on using therapy-guided LSD for severe, refractory PTSD Needs more research Any role of brain mapping in depression Not yet clinically useful would your use an SSRI and wellbutrin for 6 months This can be done Would you augment and SSRI before switching to an SNRI Usually switch classes before I start augmenting Was taught to augment Pts well-managed on their current SSRI with 75mg Wellbutrin daily at noon PRN to help w delayed ejaculationorgasm. Thoughts Yes, this can be used! I do see psychiatrist add wellbutirin or other rx on your list of other antidepressant rx to SSRI. Is there any study about the effectiveness of the combination Adding Bupropion has the best basis in the literature How about using SSRIs with SNRIs Combination usually not necessary Regarding response to SSRI do we consider it a failure to respond after reaching the full dose or after 4 weeks at any dose No response after a month at the usual dosage Does the scoring matter in the 2 question questionnaire. A yes to either one is a positive screen Do all patients with a diagnosis of depression need a PHQ9 at every visit regardless of the reason for the visit It is good for following response. Does QIDS replace the PHQ for the follow up visit Can use either I have seen patient on 2 SSRI, is that correct No, technically that is poly pharmacy. Try to avoid how would a CMP help with depression Detect reanal or hepatic dysfunction what about folic acid for augmentation of SSRISNRi Can try! Sometimes helps Is Cytomel preferred if patient has hypothyroidism (and on Levothyroxine) And would you need to monitor thyroid panel more closely If patient is on levothyroxine, would not start Cytomel. Could try increasing levothyroxine a bit to augment If a patient is on an ssri, can you discuss the safety of adding duloxetine Prefer to switch to duloxetine rather than add on if a patiuent has hypoNA with one SSRI, is there less risk with trying a different one Can try alternate SSRI. If it reoccurs, switch to different class of antidepressant Do you need to monitor thyroid function if you try augmentation w Liothyronine No, you follow it clinically, with heart rate, etc. (But do check thyroid before starting Cytomel) can you perform a follow up PHQ9 to assess response Yes! how long do u augument with another medication for depression Not clear from literature. I just continue it as long as the patient is on antidepressant If a patient wants to change to a different SSRI, must there be a break between meds No, can switch directly over Does using a more relaxing SSRI have better outcomes when using it for treatment for Anxiety Surprisingly, no. All work well QIDS vs PHQ to assess response to treatment Either, your preference PHQ9 score over 20, you start antidepressant but referral to BH takes a month. What do you do no Start a $4 SSRI can we combine SSRI and SNRI or can we use SSRI w other antidepressant meds Try not to combine antidepressants, if possible what antidepressant would you recommend for patient concerned about weight gain Avoid tricyclics and mirtazapine. All others equally good For how long do we use treatment augmentation Not clear from literature. I continue it as long as I continue the antidepressant. Is it the cancer medication or coping with chronic illness that leads to depression in people on anti ro plastics Can be both. Depression can be multifactorial. We should still treat it.

Behavioral Medicine II How do you differentiate between hypomania as part of Bipolar or antidepressant side effect such as with Cymbalta IF patient becomes hypomaic on antidepressant, consider bipolar diagnosis Are providers utilizing Lithium to assist with dementia I was told that there was a study that showed some possible assistance No, risks greater than benefits What about Cymbalta for anxiety Does it work Yes! Do you consider depression and anxiety as a spectrum disorder Two spectrums that frequently coexist and intersect. what medication should I take prior the board exam Recommend a single malt scotch AFTER the exam what about choice on benzo you did not specify choice Decide between short or long half-life benzo. Otherwise, all are similar Let's go get kicked out of an Applebee's After you! If patient has social anxiety but not daily (performances 1-2xwk), what is the preferred agent if patient does not want to commit to daily med like SSRI If the social situation is infrequent and predictable, can use benzodiazepine prn is there a correlation between social anxiety disorder and irritable bowl syndrome Anxiety can affect GI symptoms What’s best to add in patient on duloxetine for fibromyalgia and with new symptoms of Maximize duloxetine to 120 mg a day, consider augmentation Continue mood stabilizer indefinitely. If euthymic and non psychotic and on mood stabilizer, should try to how long do you treat bipolar before you start taking off medications stop the antipsychotic What medicine can you use for anxiety with least sexual side effects Benlafaxine and mirtazaline have less (fewer) sexual side effects (but not zero) Of all SSRIs, which is most likely to cause hyponatremia and why Any can cause it As SSRI can assist with depressive symptoms in bipolar when Lithium is used, can SSRI be used with other mood stabilizers Yes! What about alcohol abusers and medications Be careful and judicious with controlled substances Is the loading dose required for those who are not manic but have bipolar No, can gradually titrations up dose if not acutely manic for valproic acid, do you use actual weight (300 lb patient) or adjusted Actual (the formula is not that precise) Could only atypical anti psychotics be started for bipolar patients without mood stabilizers During. mania, but not recommended as monotherapy. Should also start mood stabilizer Is it necessary to check levels for the anti-convulsive medications if using as mood stabilizer Yes, it’s recommended When should you avoid antidepressants when treating bipolar disorder Start mood stabilizer first for a week or two, then add antidepressant Avoid lithium. Contraception is a must while on mood stabilizers. If bipolar patient wants to become What is a preferred mood stabilizer for family planning pts pregnant, needs a careful riskbenefit discussion Wats your thought on hydroxyzine prn for anxiety Can be useful on a prn basis How about Seroquel for bipolar treatment and perhaps even as as monotherapy Hesitate to use as monotherapy. Would start mood stabilizer. Antidepressant of choice to augment lithium in depressed bipolar 1 Just avoid fluoxetine due to the long half life. What mood stabilizer do you switch a pregnant patient to if they are on lithium Often use antipsychotics if mood unstable during pregnancy, then restart mood stabilizer after delivery

Behavioral Medicine III so if a pt has tic and does not improve with nonstimulant.. whats next step Can still try stimulant, watch for development of new tics Can you combine Strattera and an SSRI for depression Not recommended OCD has rituals, obsessive concerns, whereas GAD is more worrying about not making a mistake, not some symptoms of GAD and OCD sound the same.How can we differentiate worrying about being perfect OCD incrementally increase SSRIs to reach high dose to monitor for side effects, or start on high- dose right away Slowly titrations up as tolerated What is the cv defect that the ecg identifies when screening in ADHD Propensity for arrhythmias Is there a minimum age for diagnosis of bipolar Be skeptical if under 12 years old. Consider Dx of Dysruptive Mood Dysregulation Disorder (DMDD) What is most effective nonstimulant drug Strattera vs Bupropion Atomoxapine (Strattera) do you prescribe adderal with vyvanse together like 2 stimulant to treat Occasionally use immediate release Adderall in early afternoon if Vyvanse wears off too soon. if transitioning between nonstimulants (if one is not effective) for adhd, how long you wait Can switch right away any difference in choice of tx for different types of ADHD Among meds, stimulants 1st choice what is the youngest age that one can prescribe Stratterra for Usually 6 yo Does strattera have any preference for benefit regarding inattentive vs attentive type No can you use a stimulant and nonstimulant for ADHD Would avoid polypharmacy what about trial wnonstimulant drug trial if getting neuropsych testing for ADHD too onerous my attending suggested this Could certainly try bupropion How often should urine drug screen be ordered for adults on a stimulant Does all stimulant give positive amphetamine Yes it will usually show up in drug screen. Schedule once or twice a year at random times can you use long acting and short acting amphetamine at the same time Long acting in AM and short acting at lunchtime.. Yes why autism and ADHD are more prevalent in the US than Europe Different sensitivity and awareness of the diagnosis How can we differentiate between anxiety and ADHD, especially when newly diagnosing in adults This is where the testing will help amphetamine derivative medications (getting more and more patients on high dose medications in their 40s) If concerned, do the cardiovascular testing. How often should an adult ADHD patient be seen in the office Recommend q 3-4 months What is the earliest age to Dx ADHD Usually picked up when child starts school what is the course of ODD as one ages Can develop into conduct disordering not treated Why do alpha 2 agonists work and do you have a way to remember how the different types of Don’t really know the exact mechanism. Just have to memorize the drugs Have read that Vyvanse depletes body’s natural serotonin stores & can worsen depression long- term. Thoughts Not a major concern or problem. How effective is bupropion for adult ADD Not as effective as the stimulants what labs should we check before we start stimulant for a child.. CMP Labs not necessary. Would check BP after starting It is teccomensed that the patient sign a controlled substances agreement, with options for random drug do you use random urine drug screens for patients on stimulants screens have you seen guanfacine work in adults for ADHD Yes do you repeat vanderbilt testing after starting treatment Yes. It can help document response I have heard that you have to have had adhd as a child to be dx as adhd as an adult. Is that true You should be able to find symptoms that extend back into childhood, but this can be difficult at times how do you choose with rx to start first for ADHD Personal preference familiarity. No way to predict which will work But really, is the door locked Why did you have to ask me that!!! can u take adderell even if u dont have adhd for studying No. This is how abuse dependence can start What are nonmedication therapeutic options for people to overcome forgetfulness and inattention Behavior and studying modification based on what the psychological testing shows is memantine useful adult ADHD No So is the most common co-occurring psychiatric illness in ADHD is learning disability or ODD In boys, it’s ODD. The learning disability is for all patients Why do STIMULATANTS improve hyperactivity The theory is that they stimulate the inhibitory behaviors to a greater extent than the impulsive pathways

COPD, Lung Cancer, OSA, and how would the exam ask about spectrometer measures FEV 1 and FVC are important is there a study to see if because of LDCT lung screen, pt would continue to smoke because they get the reassurance of no cancer Interesting question Isn't Wegener's now called GPA Yes, granulomatosis with polyangitis, or GPA. The treatment for sarcoidosis - only hilar adenopathy is observation, but in the question the patient have BL perihilar infiltrates-- dosent this patient need systemic steroids If symptoms sarcoid. if only treat when symptomatic, why follow and do ACE levels Follow to ask about symptoms in a systematic way. ACE level at beginning, not very useful for following. How do you follow asymptomatic sarcoid pts Just for symptoms Symptoms what type of changes on CXR would be concerning for lung cancer how often to follow Low dose ct should be read by a trained radiologist and follow those recommendations Is NAC the same that is used as antidote for Tylenol overdose This is an OTC supplement form, in a capsule for oral use. Many pts are replacing smoking with hookah what is the risk of cancer with this past time Unknown, but probably similar risk What about using an autotitrating PAP machine Excellent tool, auto titrations are useful If stop bang score is high.. do you recommend sleep study before surgery Sleep study good first test. ENT consult if you suspect anatomical problem. if pt with OSA has more than 10% weight loss what do you do- retest them or discontinue Good idea, especially if mask not fitting. Any role of chest physio in COPD management No role for chest pt Does medicare cover sleep studies Also is home study reliable Yes and yes we order 6 minute walk tests to qualify for ambulatory home O2 as we are discharging our COPD patients. Can you comment Good appropriate practice So for low dose CT, say yes to screening for exam purpose There was a recent update not ot routinely offer it. What do you say I have not seen the update to not offer ct screen. I think they are stressing “shared decision making” Is there any role for SAMA for exacerbation of COPD or maintenance therapy, or SABASAMA LAMAs are better than SAMAs for copd, they are longer acting, and have higher affinity for the receptors. If combination too expensive, then use a SAMA. acute exacerbation, hospitalized, yet LAMALABA are non formulary; stop while in hospital or have bring in own We use duonebs and steroids and antibiotics while in hospital. Resume LAMA LABA first discharge day Do you have a preferred LAMALABA (not considering insurance coverage) Android easiest to use is CCQ test same as CAT for COPD No, CCQ and CAT test questions are different for the Exam, we should be repeating spiromerty annually Yes is a pt quit 14 years ago and they need lung Ca screening. how many years do you repeat CT chest Until quit 15 years ago why COPD pt is OK to use albuterol inhaler daily but not in asthma Different conditions, research evidence If pt comes to you on Theophylline and controlled, do you change treatment No, if no side effects. Monitor level, aim for 5-12. Putting COPD patients on chronic ICS doesn't decrease exacerbations but it does put them at higher risk for pneumonia Use for sub populations of high eosinophilia What is the role for overnight pulse oximetry in the dx of OSA. Currently based on apnea’s, but pulse ox may be useful what is the OTC rx for COPD failed to repsonse to 3rx NAC if a smoker smoked 4 ppd x 8 years, does that meet criteria for Low Dose CT scan to meet USPSTF recommendations for lung cancer screening. Basically it is according to Pack Years 4X8 =32 Yes, we go by pack years, 30 or more do u still do annual low dose ct if smoking less than 30 years Not if less than 30 pack years, which is 1pack per day for 30 years. NAC. n acetyl cysteine Yes HOW LONG CAN WE GIVEN THE ZITHROMAX 1 year is longest study To clarify, are LABAs safe as monotherapy for acute COPD symptoms Ok, but usually used in combination. LAMAs first. are there any official statements on vaping as compared with smoking for copd attainment Vaping not recommended by cdc can we add NAC for COPD in the beginning instead of after 3rx failure Usually after they have had frequent exacerbations After Dx of COPD, How often is recommended to do Spirometry Annual

Fracture Care in Family Medicine if newborn with clavicular fx how do you places in a sling and how long In newborn, place in figure eight or if no displaced then no treatment required. How is answer choice A not considered FALSE Malleolar pain is not required based on the definition Malleolar pain or pain in the region of the navicular or proximal fifth metatarsal is the first requirement of provided. the Ottawa Ankle Rules how long do we use bisphosphonates to treat osteoporosis 5 years and stop or continue for upto Five years total exposure then stop. E.g. Two years on, two years off, three years on then have to stop or 10 years 2 year drug holiday switch to different class of agents. When is surgical intervention recommended in VCF Can consider kyphoplasty or vertebroplasty for unremitting or uncontrollable pain. Yes. DEXA. Maybe MRI. If younger pt found to have nontraumatic vertebral compression fx, do you do any workup Obviously you'd already have radiographs to make the diagnosis. wat about male with vertebral fracture, do we get DEXA Yes. No bisphosphonate after completing a total of five years of bisphosphonate therapy (whether continuous so to clarify, no bisphosphonates EVER after 5 years of treatment or interrupted). why doing bone density in fragility fracture. we can assume of diagnostic of osteoporosis and start You will need the actual bone mineral content to follow up progress at two years to determine efficacy of to treat. treatment As soon as the patient is comfortable to get the DEXA. They will need to lay still and lay flat (supine) for How long after fracture should you wait to get a DXA about 3-4 minutes Long arm splint; posterior splint from mid to upper humerus down to provimal hand with the elbow in 90 In what position should we splint a child who can’t move a broken elbow degrees of flexion Evaluate with X-rays if concern for fracture, and if concern remains then splint for two weeks and then re- How urgently does the patient need to be referred for ortho eval with FOOSH injury evaluate. If still concern for fracture and radiographs remain negative then either MRI or splint and refer. scaphoid fx.. not seen on xray at repeat in 2nd week later. get CT next I would question my diagnosis but if still concerned about fracture then a noncontrast MRI of the wrist why restrictive lung dz in older women with vertebral compression fxs Thoracic kyphosis leads to mechanical restrictive pulmonary disease Buckle Fx what if parents unreliable Then you have to consider short arm cast or splint and follow up in three weeks

Musculoskeletal Medicine how to you work up an 80 year old with urinary incontinence who develops low back pain do you It probably depends on the time frame for progression, but I would certainly consider early imaging for go for imaging lumbar myelopathy. What is effective dose of cymbalta for chronic back pain 60 mg per day Won't steroid injections for plantar fascia lead to rupture Low risk of rupture with these injections, but I would counsel patients on the risk any benefit for augmented soft tissu mobilization (ASTM) therapy for carpal tunnel sundrome or Yes, there is benefit from these therapies. They are just not the preferred therapies but can be done in other tendinopathies conjunction with eccentric strengthening for chronic tendonopathies. It has not been shown to be effective beyond three weeks, and patients often are challenged in using it For plantar fasciits why no night splint for >3 weeks even for three weeks. This is a technique used when surgical treatment is considered as fewer complications than surgical capsular distention by injection help or no for adhesive capsulitis manipulation. Autologous blood is whole blood, PRP is plasma only so it involves the whole blood to be spun down to What is difference between autologous blood injection vs PRP obtain the plasma. Use of nitro patch in tendinopathies Yes. Eccentric contractions involve the phase of muscle contraction where the contracted muscle is being lengthened. What are eccentric strengtheningcontractions In the concentric phase the contracting muscle is being shortened. how do you define non inflammatory for overuse injuries there does seem to be signs of inflammation with them Cellular examination by biopsy demonstrates no inflammatory infiltrate. is home stretching plan effective for LBP This would fit into a home exercise program and is beneficial to reduce recurrence of a LBP flare. Can you overlap Duloxetine with tramadol Is there interaction Yes and yes. Need to be cautious. how long will you treat with duloxetine and what daily dose limit A minimum of three months. Goal dose is sixty mg per day. can you explain tarsal tunnel syndrome pathology, and treatment. where would the injection be Injection is into the carpal tunnel via a volar approach. located Pathologenesis is related to compression of the median nerve in the carpal tunnel. What is would be dose of Tramadol that is effective Highly variable. I usually start with fifty mg every six hours and titrate from there. there was a mention that Tylenol is not effective for chronic pain but it is still recommended for It is not effective for acute LBP bit IS effective in chronic LBP do you get a xray of back for pt's who have RISK for prostate cancer (age, race) but no official urinary symptoms No. Risk factors for prostate cancer are not a red flag LBP- what about patients who are already max treated on NSAID without relief Add in non drug therapies. Consider adding in duloxetine if this is becoming chronic. is Gabapentine effective for chronic LBP No. Not recommended. if there is very little evidence for opiates in chronic back pain, why do you recommend it for chronic Tramadol only is recommended for consideration. back pain Little to no benefit for methadone, morphine, oxycodone, hydrocodone No, unless they are administered by transformational epidural steroid for acute radical are symptoms with are steroids EVER the correct board answer for LBP evidence of herniated disc causing nerve root compression. Is there evidence for difference between home versus formal PT Generally the answer would be "no" median nerve This is the nerve involved in carpal tunnel syndrome No. The patient with cauda equine syndrome will have repeated episodes of incontinenece, but one Do you consider just one episode of urinary incontinence as a red flag and need imaging significant episode of urinary retention in the setting of LBP would be a red flag. How come NSAID are beneficial while steroids aren't (both are anti-inflammatories) I don't agree that NSAIDs are primarily anti-inflammatory in effect. NSAIDs are potent analgesics. What are the side effects of combining NSAIDs with skeletal muscle relaxers Side effects of each drug without benefit in added analgesia Cold compresses vs heat for musculoskeletal pain Cold helps with swelling and acute pain. Low level heat is generally more effective for chronic pain Gabapentin is not recommended for use in LBP by review of evidence from the American College of What is gabapentin's role in back pain in the guidelines Physicians or the North American Spine Society. This is a new recommendation in the past two years or so. Lot of ERs are treating with steroids for low back pain, any thoughts on this This is wrong, even though it is being done. You've got the newest and best information. how about the patient(s) who have been on opiates for many years and are reluctant to try This is complicated and would include a MH provider to help explore the patient's concerns which can be anything else out of sync with clinical evidence Just to be clear, is it not recommended to use NSAID with muscle relaxer for Acute LBP Correct

Selected Issues in Women's Health In case of a post menopausal woman with Uterus on topical estrogen replacement, what form of Progesterone is suggested. Typical regimen – 10-12 daysmonth; e.g., 10 mg medroxyprogesterone If patient becomes pregnant with IUD in place, do you take IUD out or leave it in Remove is recommendation Are certain lubricants or moisturizers contraindicated or decrease chances of getting pregnant in females seeking pregnancy The only issue would be if the lubricant contained a spermicide. what is the difference between progesterone containing IUD and containing IUD The progesterone IUD is a hormonal system. The copper IUD is non hormonal. In what form do you prescribe progesterone when vaginal estrogen dose > 5 Typical regimen – 10-12 daysmonth; e.g., 10 mg medroxyprogesterone if history of breast cancer and atrophic vaginitis, will low dose hydrocortisone help prn for No No. Estrogen Tx for menopausal Sxs & breast cancer risk Use a non-hormonal treatment option as outlined in the table in the presentation. How about TAH due to cervical cancer Do you do pap Yes - for 20 years annually How do you collect the HPV alone Bloodcervical Cervical just to clarify for Atrophy of vagina we place estrogen tab intravaginal One can use a vaginal ring, vaginal tablet OR vaginal cream. Any association with IUD and influencing GYN cancers No Besides not following guidelines, why are ObGyns doing paps yearly on all their patients No idea. do you still perform a pelvic exam without a PAP after the age of 65 for physicals You could. The AAFP does not recommend routine bimanual examinations. What about PAPs in patients who underwent Subtotal Hysterectomy If they have a cervix, they should be screened for cervical dysplasia. Screening pelvic examinations, except for the purpose of performing cervical cancer screening at recommended intervals, have not led to reduction in mortality or morbidity, and expose asymptomatic women to unnecessary invasive testing. Noninvasive options to screen for sexually-transmitted infections are now available as alternatives to endocervical cultures. Screening pelvic examinations also add I understand yearly pap is unnecessary after 65 or no cervix, however, women are requiring yearly unnecessary costs to the health care system, included expenses from evaluations of false-positive findings. pelvic exams, correct These pelvic exams can even lead to unnecessary surgery. for PO estrogen and they have a uterus, would you add Progesterone YES how do you get irregular bleeding with prog. only pills when there isn't estrogen to cause It creates an atrophic endometrium. So is there a length of time that a patient shouldn't be on DMPA 5 years 10 years There is not. What about women that you can’t get an EB (eg stenotic os)...what test next D&C can you use black cohosh for menopausal symptoms No data it works. In postmenopausal bleeding with TVUS normal do you still do EMB Yes No Do I need to stop local (vaginal) estrogens in women post stroke Would use the Vaginal tablet - 5ug

Sports Medicine can you repeat the different positions of paresthesias to differentiate handlebar vs. ulnar Ulnar n. Guyans canal = palmar or volar ulnar digits neuropathy please Ulnar n. Cubical tunnel = dorsal ulnar digits If 48 hrs post injury pt still has a bad HA why is it rec to start light activity Slide states "below Then you may have to wait until the HA improves to the point that they can do some light exercise and not symptom threshold" cause a worsening of their HA When pain free and full range of motion and return of at least 90% strength (compared to uninjured After shoulder re-location - when can you return to play shoulder girdle) Flex ion ADuction Internal Rotation what does FAdIR and FAbER test for Flex ion ABduction External Rotation Usually an elbow brace to prevent repeated elbow flexion in cubical tunnel best treatment for ulnar neuropathy Wrist brace or padded cycling gloves or both if affected in guyans canal Do taping ankles and bracing ankles have the same risk reduction in ankle injuries Generally "yes" You're uncomfortable with care, worsening if symptoms when you would expect them to improve, What are the indications to refer to concussion clinic after an episode SRC prolonged symptoms beyond four weeks (kids 5-12 years) or greater than three weeks for all others Are we allowed to clear them to return to sports after a SRC or do they have to be cleared by Each state has its own requirement for who can clear athletes; but in all states Family Medicine physicians sports medicine can return athletes to play post concussion how do you do a stat rectal temperature on a sports participant in the field to diagnose a heat illness doesn't sound realistic to do a rectal temp. Just do it. athlete had splenomegaly, imaging was done at diagnosis, do I then reimage to determine that the The problem is that imaging is unreliable. Yes, you would probably need to repeat imaging, but if no change splenomegaly has resolved then you have to consider whether athlete just has a large spleen. Demonstrate one or two techniques for reduction of shoulder dislocation on the sidelines. See above What maneuver to use for reduction of anterior shoulder dislo See above Will we need to know the various elements of the 14-Element Cardiovascular Screening Checklist Generally yes, specifically no. Have a general idea of historical questions and possible exam findings There is always a question on the 'female athlete triad'. Any pearls or recommendations See supplemental slides How to reduce anterior shoulder dislocation See above for stress fx in runners, after xray is negative; do you get bone scan or CT Usually MRI but can consider treating clinically as well with reduction of activity and clinical follow up how to reduce anterior shoulder dislocation Multiple options which I would recommend you look up and memorize one.

A Review of the Upper GI Patient family members often want to be checked for h pylori if pt tests positive. Are there recommendations for this I've only done it if symptomatic or if fam hx of gastric ca. There is NO recommendation to screen family members for H. pylori. In patients with alarm symptoms, do you get an EGD first or only if symptoms continue after therapy for 4-8 weeks EGD should be done Is there epigastric tenderness Has the patient been on acid suppression meds and NOT responded

Functional dyspepsia AND PUD are associated with epigastric tenderness. GERD is NOT typically associated PUD and GERD can present similarly. This confuses me on when to get urea breath test. with tenderness. AGA Choosing Wisely

"For pharmacological treatment of patients with gastroesophageal reflux disease (GERD), long-term acid if you have a new pt comes with long term use of PPI because heshe was told to have GERD. suppression therapy (proton pump inhibitors or histamine2 receptor antagonists) should be titrated to the Would you try to wean off the pt if so, how lowest effective dose needed to achieve therapeutic goals." Why straight to the urea breath test and not serology You want a test for ACTIVE INFECTION. Folks who have been taking PPIs for a period of six months might consider tapering down their dose instead of stopping cold turkey. Try to reduce the dose by 50% every week. Once the patient is on the how do you titrate down ppi lowest dose for one full week, you can try stopping the PPI. >= 60 - EGD is recommended for new onset. If you suspect PUD, wouldnt you just send them to GI to get a scope (most importantly), and they would test for H. Pylori. <60 - test for H. pylori. IS long term PPI and H2 therapy the same as life time what is the end point Symptom control. what is on demand PPI, frequency On demand = I have heartburn and I take a Pepcid or Prilosec. ppi and plavix should patient still take it Yes. any screen test EGD or us with FHX of stomach cancer which is more frequent in Asia same question with pancreatic cancer. There is not a published recommendation AGA - Choosing Wisely

"For pharmacological treatment of patients with gastroesophageal reflux disease (GERD), long-term acid how do we treat patient with gerd whosse symptoms persist longer than 8 weeks use of PPI as suppression therapy (proton pump inhibitors or histamine2 receptor antagonists) should be titrated to the long term use has side effect risk profile lowest effective dose needed to achieve therapeutic goals." So the January article highlights Functional Dyspepsia "epigastric discomfort without evidence of organic disease found during an upper endoscopy." In these situations - the preceding working dx (before endoscopy) is typically gastritis or PUD given their is tenderness on the exam. As per our discussion, if we are thinking PUDgastritis based on our history and physical - then yes a test for H. pylori ACTIVE infection is indicated. Previously, in terms of endoscopy - the rec was if > 55 (and particularly is ANY Alarm sxs) an invasive eval (EGD) be undertaken rather than non-invasive H. pylori testing be done in the contestst of starting treatment. The new rec raises that age to 60. "Endoscopy should be performed for all patients 60 years and older with at least one month of dyspepsia please could you speak to the AAFP journal last year that recommends hpylori testing symptoms." is there proven evidence for dementia and ppi There is NOT. How long does a patient have to be off PPI before doing Urea breath test Minimum of 2 weeks, preferably 4 weeks. The 2017 ACGCAG guidelines recommend upper endoscopy be performed for all patients 60 years or older who present with at least one month of dyspepsia symptoms. The age threshold may be lowered in patients with a higher risk of upper gastrointestinal malignancy (e.g., patients of southeast Asian descent). Endoscopy should not be performed routinely for patients younger than 60 years. This recommendation is based on the low prevalence of malignancy in younger patients and cost-effective economic modeling.

Are these GERD guidelines for all ages I was taught older pts that have new GERD symptoms to get Moayyedi P, Lacy BE, Andrews CN, et al. ACG and CAG clinical guideline management of dyspepsia. Am J EGD. Gastroenterol. 2017;112(7)988–1013. is there any difference between dyspepsia and GERD Functional dyspepsia and GERD are synonymous. Generic g=followed by the MOST COMMON trade name e.g. - A. Azithromycin (Zithromax) in board questions, will meds be given by generic name or brand name B. Famotidine (Pepcid) Oral or inhaled steroids for eosinophiic esophagitis Oral So your are thinking surveillance for Barretts, I presume

Consider screening in MEN Chronic (> 5 years) andor frequent (weekly or more) symptoms of GERD AND Two or more risk factors for BE or Esophageal Adenocarcinoma (EAC)

Caucasian race Age > 50 years Central obesity (waist circumference > 102 cm or waist-hip ratio (WHR) > 0.9) In patients on long term PPIs (more than three years), at what point do they need an EGD for Current or past history of smoking surveillance Confirmed family history of BE or esophageal adenocarcinoma (in a first degree relative) If you are thinking PUD or Functional dyspepsia (these patients have epigastric tenderness) - most certainly I have a very international population, should I not check for H pylori after PPI failure before look for H. pylori via test for active infection. If it is GERD based on H and P (no epigastric tenderness) - the manometry current published recommendation is to NOT test for H. pylori. disease found during an upper endoscopy." In these situations - the preceding working dx (before endoscopy) is typically gastritis or PUD given their is tenderness on the exam. As per our discussion, if we are thinking PUDgastritis based on our history and physical - then yes a test for H. pylori ACTIVE infection is indicated.

The 2017 ACGCAG guidelines recommend upper endoscopy be performed for all patients 60 years or older who present with at least one month of dyspepsia symptoms. The age threshold may be lowered in patients with a higher risk of upper gastrointestinal malignancy (e.g., patients of southeast Asian descent). Endoscopy should not be performed routinely for patients younger than 60 years. This recommendation is based on the low prevalence of malignancy in younger patients and cost-effective economic modeling. SOR C

AAFP article 11520 "For pts <60, a test and treat strategy for H. Pylori is recommended before acid Moayyedi P, Lacy BE, Andrews CN, et al. ACG and CAG clinical guideline management of dyspepsia. Am J suppression therapy." Gastroenterol. 2017;112(7)988–1013. The only two drugs safe in the elderly are acetaminophen and colace. What about PPI in elderly...it's onthe Beer's list. Always RiskRisk and try to get them to H2 blocker. how often should you check b12 and magnesium on chronic ppi therapy Annually

A Review of the Lower GI If the patient has no red flags but does not report improvement with defecation can you still By definition - IBS is diagnosed using the ROME IV criteria. There is a continuum of functional GI disorders, diagnose IBS if pt co "years" & no red flags of course. And the key with functional disorder diagnosis is no evidence for organic disease. How long do you use antibiotics for IBS 14 days Loperamide can be used in travelers diarrhea(TD). No other nonantibiotic treatment for TD has significant Do you ever recommend anti-diarrheal medication guideline or clinical trial support. Vaccination should not be initiated for infants aged 15 weeks and 0 days or older because of insufficient data on safety of dose 1 of rotavirus vaccine in older infants.

The rotavirus vaccines are only licensed at these specific ages, so were not studied in older infants and toddlers.

Also, although the risk is low, a small risk of intussusception after getting the rotavirus vaccine is thought to why is the limit time frame of rotavirus for first and last dose increase with increasing age of the first dose. Cost Why offer fobt when cologuard is more accurate for stool testing in colon cancer screening Lives saved is essentially the same regardless of screening tool selected. Why do some guidelines suggest starting colon cancer screening for African Americans at age 45 if African-Americans are more likely to develop CRC at a younger age and to be at a more advanced stage they have equal rates of occurrence when diagnosed. NO DRE is NOT a screening test for CRC.

The patient is provided with a packet of three cards and MUST undertake dietary modifications for up to a To get the gFOBT, you do the DRE, right week before testing. There is NO repeat in 1 year on screening colonoscopy.

Please refer to slide in handout with table which reviews this. The slide is entitled "Guidelines for Follow-up Which polyps pathology needs to repeat colonoscopy in 1 year, 3 years and 5 years Surveillance Colonoscopy" (Slide # 84 - Lower GI) ) An advanced adenoma is defined as an adenoma that is 10 mm or larger, has villous elements, or has high- what is the definition of advance adenoma in CRC screen grade dysplasia Rifaximin what abx used for Irritable bowl syndrome Neomycin Is gluten free diet really effective YES! Studies of the rotavirus vaccine have shown that it can prevent about 74% of rotavirus infections. More importantly, it can prevent approximately 98% of severe infections and 96% of hospitalizations from Why is rotavirus common in USA if vaccinated against rotavirus. ANTIBIOTIC Prophylaxis is NOT routinely recommended.

Self-treatment for travelers’ diarrhea has become so predictably effective that most physicians no longer recommend drug prophylaxis against diarrhea except for certain high-risk travelers or when the trip is deemed critical. You might consider prophylaxis with either Pepto-Bismol or antibiotics if you will be traveling short-term (less than 3 weeks) and cannot afford to have your trip interrupted, or travel plans altered, because of illness. You might be, for example, a business person, diplomat, musician, or athlete who can’t afford to miss even 1 hour of an important meeting or event.

Or, you might have a medical condition that would be adversely affected by any additional illness. Medical conditions warranting consideration of prophylaxis would include cancer, AIDS, severe inflammatory bowel disease (colitis), kidney failure, and poorly controlled insulin-dependent diabetes. Also, if you have peptic ulcer disease and take a stomach acid-reducing drug (e.g., Zantac, Pepcid, Prilosec, Nexium, or Protonix), your risk of travelers’ diarrhea is increased. Consider taking the anti-ulcer, stomach-coating drug Carafate (sucralfate). This may reduce your risk of diarrhea because Carafate has antibacterial properties.

Pepto-Bismol Taking Pepto-Bismol (bismuth subsalicylate) will reduce your chances of getting travelers’ diarrhea by about 65% (compared with 90% efficacy for antibiotics).

Two tablets (or 2 tablespoons of the liquid), 4 times daily. Take with meals and at bedtime. The tablet form of Pepto- Bismol is as effective as the liquid preparation, and the tablets are easier to carry. The downside taking medication four times a day is very inconvenient for most travelers. Children’s dosage Pepto-Bismol may be used by children older than 3 years. They should use one half the adult dose. For using Pepto-Bismol in a child under age 3, consult your pediatrician.

There was a recommendation for zithromax in travelers diarrhea , short course for prophylaxis,in Note 2 tablespoons or tablets of Pepto-Bismol have the salicylate content of about one adult aspirin tablet. Pepto- the AAFP questions and the journal Bismol is most effective when taken with meals to allow the drug to come into immediate contact with the

Management of Chronic Pain What is your opinion on massage as treatment It is a reasonable adjunct In states where it has been studied, opioid use has decreased as marijuana Rx increased Are there risks of use of marijuana with combined use with opiates The combination is likely more sedating and disorienting Yes should random drug screens include checking that patient is taking the medication prescribed Looking to see if or is taking the drug that is prescribed, and not taking illicit drugs what is the reason to use Naloxone or combination for tx of opiod abuse as they are long acting The naloxone component is inactive if the drug is used as prescribed opioid rx themselves If the drug is adultered and injected, the pt will exhibit withdrawal Can you talk a little about fast metabolizer's when it comes to the UDS. We always send out If pt is taking the medication chronically it should show up on GCMS study, either as the parent drug or confirmation but not uncommon for patients to swear up a down they took the medication but usual metabolite UDS and confirmation is negative. It would be reasonable to contact the lab and see if there are other reasons for the neg results Please make comment on Tylenol usage for chronic pain. Most of chroinc pain as from arthritis, Tylenol is effective for chronic pain, especially pain from DJD etc. One of the speaker said Tylenol is no longer effective for chronic pain or OA Max Doses- healthy adults 4 Gday, elderly 3 G, liver disease 2 G do prescribed ADD meds come up on drug screens as positive for amphetamine Yes ,they usually do

Managing Common Cutaneous Diseases for psoriasis what corticosteroid potency do we use Start medium potency in most areas, and titrate up as needed Is it true that the worse Vitiligo gets, your risk of mortality increases I am not aware of that Congenital Nevus has up to a 10% potential to convert to Malignant melanoma. What is the recommended management for these congenital levi. Watch it closely or excise it I have never seen it used Can we still useget cantharidin solutions for molluscum It is available on the internet Do you have to treat molloscum in adults in nongenital areas Since the lesions are infectious, most people treat them Why was HPV wrong on the molluscum question HPV doesn’t cause lesions with umbilicated papules No, it is different Intertrigo can be due to a combination of bacterial and fungal agents is tinea intertrigo same as Erythrasma is a specific bacterial (prob ) infection For rosacea ivermectin 1.87% is available OTC at tractor supply stores as a de-wormer for horses. Costs $2 for a large tube. Thanks What is the ugly duckling sign It refers to an atypical nevus in a pt with several atypical nevi that looks different from the other nevi acnes is becoming quite resistant to topical antibiotics other than BP Why not use topical antibiotics as monotherapy Many pts do well with topical Clinda alone esp Current recommendations from multiple sources are not to use these other topical antibiotics as with poor insurance coverage with other agents. monotherapy It is an anti-androgen, decreases sebum production what dose aldactone for It tends to grow breasts on men how long would you continue clobetasol, high potency, 2 weeks and take a week off, and restart Depends on what you are treating I would be very surprised if this happened, as this tool is almost exclusively used by dermatologists and not would exam show pictures seen via a dermoscope commonly taught in GM residency programs if you drain a furuncule do you need to give abx do we need to know how to tell if a lesion is ready If fluctuant with discomfort should be drained for I&D If large, fever, or surrounding erythema an I&D is indicated What is Norwegian scabies Widespread crusted scabies- seen in immunodeficient patients- common in advanced HIV We used to do androscopy - used acetic acid and a magnifying microscope to look at the genital area - this is no longer done ln male is there any test to check HPV other than visual exam I No Do you use amoxicillin for acne I have seen dermatologists do this. are both antibacterial and anti inflammatory, and are are drugs of choice Good question Children spread this around by touching, and get lesions all over It would be a greater concern if there are no lesions elsewhere on the child,and other factors also point to Would comment that Molluscum in genital area on a child is worrisome for sexual abuse. possible abuse Treat with analgesics at onset of pain What is the "right timing" for postherpetic neuralgia treatment Consider nerve blocks if severe, usually if analgesics not helping after several weeks of Rx Need to wait at least 8 was after zostavax to give Shingrix if a pt got zostavax 2 years ago, when or do they need shingrix do we need to wait a period Zostavax response tends to fatigue after 5 years, so good idea to give Shingrix at that interval between the 2 types of vaccines (zostavax and shingrix) See CDC website for details Yes Pt has had Zostavax but wants newer Shingrix, ok to give See CDC website for details

Maternity Care I What about using ginger tea Ginger in ANY form. what if the 1hr OGTT is >190, should we do 3h OGTT even if we know 90% will reusult abnormal Most will not. what is the Methotrexate for in the early bleeding OB ECTOPIC Can you speak on the utility of using intrapartum antibiotic prophylaxis for GBS negative patients with prolonged ROM It is NOT indicated. Prolonged ROM is NOT an indication for treating Mom with antibiotics. What about finding GBS on urine culture in NON-pregnant woman. Do you treat and if so with what and for how long No Early-onset neonatal sepsis (EOS) has been variably defined based on the age at onset, with bacteremia or bacterial meningitis occurring at ≤72 h in infants hospitalized in the neonatal intensive care unit (NICU), For GBS screening, what does "early onset GBS Sepsis" mean versus <7 days in term infants Are there studies looking at high dose steroids for hyperemesis gravidarum and the risk of IUFD Steroids are the VERY last line of med used and avoided at all cost. would the exam ask when you perform cervical checks in clinic in a women who is not having No and There is no indication to do that. I thought 18h sp ROM was prolonged ROM It is. But time ROM has no bearing on GBS prophylaxis if Mom is GBS (-) There is no indication for repeat. That is easily done via Leopold maneuvers. I would only repeat an US if when do you repeat ultrasound in clinic to check head position is there any recs there was a question of presenting part that could dNOT be diagnosed on PE. re multivitamin daily during fertile yrs...I thought daily mvm use wo a medical condition (eg malnutrition) increased use of overall mortality or ca All women of childbearing age should be on a MVI with 400 ug folic acid DAILY. so for folic acid 4.0 mgday you are giving 1-3 m before, and continue first 3 month of preg only then after only prenatal vitamins is ok That is the dose if there is a history off NTD - correct. what dose of folic acid if has seizure disorder As reviewed in the slide #42 in the Maternity Care 1 lecture. can you clarify results of 1 hr GTT, slide states greater than 135 or greater 140. Either can be used as the threshold. do you screen for diabetes in mother during initial ob visit obese, family hx of diabetes.. would you do A1C or glucose tolerence testing Is is a Grade I from the USPSTF as we discussed. Screening for DM in pregnancy NEVER uses an A1c. HPV, only or mostly sexually transmitted Have heard mixed opinions. Is is the MOST common STI in the US when do pt who are varicella nonimmune get vaccinated postpartum 2 weeks, 6 weeks, or later After they deliver Quinolones in pregnancy No ACOG is making the red - Level C why such discrepancy between usptf rec and yours for iron supplementation USPSTF - Grade I Clinicians should be aware of the prevalence of infection in the communities they serve, as well as other sociodemographic factors that may be associated with increased risk of syphilis infection. Factors associated with increased prevalence that clinicians should consider include history of incarceration, history of commercial sex work, certain racialethnic groups, and being a male younger than 29 years, as well as what is "high risk" with syphilis regional variations that are well described. next steps after what was presented for pharmacologic intervention would be Dimenhydranate or Diphenhydrmine or Prochlorperazine or Promethazine. If none of that works - third line is Metoclopramide no ondansetron at all in pregnancy or Ondansetron or Trimethobenzamide. do you stop breastfeeding if mother develops active chickenpox Yes If history of orolabial HSV, but no history if genital herpes, do they need suppressive viral therapy No Only if If she has a risk factor intrapartum

EGA < 37 weeks ROM > 18 hours Do you prophylax if mom is G0 and GBS unknown Maternal temp >=38C Only if If she has a risk factor intrapartum

EGA < 37 weeks ROM > 18 hours if mom shows up in labor and GBS state unknown... treat or not Maternal temp >=38C pregnancy in 33 year old who had a pap smear 4 years ago, does not need pap on initial OB visit, correct since she was co tested 4 years ago Correct. Pregnancy is NOT an indication for pap. when a pt with asym bacteriuria, when do we do test of cure.. UA and Urine culture or just culture Culture

Renal Disease I They are subsets of glomerulonephritis Nephritis have hypertension, RBC casts, mild proteinuria - think vasculitis Can u go over nephrotic n nephritis syndicate subcategories Nephrotics are edematous, have profound proteinuria - think DM nephropathy Is 7 days of nitrofurantoin for UTI toxic with CrCl <60 Could be, especially with very low CC There are several in use, including renal US, CT angio, MRI Diagnostic test of choice to evaluate for Renal Artery Stenosis Tends to vary by institutional experience and expertise Typically much higher than 1,000 what is the CK level to dx rhabdo Usually have pos urine dip for blood, and muscle symptoms why allopurinol to help precent Aki Prevents damage from uric acid nephropathy which can happen after cell breakdown following Not that I am aware of Any pressors to avoid in AKI Dopamine and norepinephrine are used most often in our institution Sounds reasonable what level of K do you use Ca K above 6.6 Would use it for K > 5 if there are hyperkalemia associated EKG changes MAP > 65 for AKI is our goal MAPs above 65 or systolic BP above 100 (for perfusion) Keeping systolic above 90 is also wise Correction Are there specific diuretics (Acei arb lasix spironolokactone) for which one must use of ACE or ARB and all diuretics will increase Na excretion in the urine and will make the FENA equation difficult feurea equation to interpret I avoid using NSAIDS if pts have albuminuria or GFR < 60 what stage level of CKD before NSAID can not be given. If so, is tramadol the only choice for pain tx Tylenol can be given for pain, and other adjunct agents are availability prior to choosing opioids At 45, don’t start metformin, cut dose by 50% if doing OK on it can you clarify GFR and starting metformin. slide states 30 and 45 At 30, stop it. In post renal AKI due to obstructive uropathy, do we do IVF if we are concerned about hydronephrosis or hydroureter If the obstruction is due to ureteral stones, hydration may help flush them through WBC casts = pyelonephritis can you please differentiate between the various types of casts noted in urine and what is are the RBC casts = glomerulonephritis causes Hyalineprotein casts - can be normal or can be seen in various renal diseases Does AIN happen with prolonged PPI use or also with prn use I don’t know. It is not a dose related phenomenon. what is the cause for + dip stick for blood but no RBC Rhabdomyolysis (myoglobin in urine) and hemolysis (hemoglobin in urine) are the major causes what is the difference in utility between GFR AND CRCL Clinically, we use them in the same way. is there any reason to check for microalbumin in DM if they already have dx of kidney disease whethere it is mild or not Yes. The presence of macroalbuminuria (>300) predicts progression to CKD4-5 Look at the urine - if you see any more than 1-2 epithelial cells per HPF it is probably contaminated and UA positive for LE. When do you say UTI vs contaminate What is your cutoff your C&S will likely come back as mixed flora

Renal Disease II Was is the dosage for ACE 's inh for kidney protection 2.5, 5, 10... As high as pt will tolerate The CDC’srecommendations regarding pneumococcal vacation are very complicated and confusing The new thing is that normal adults over 65 do not seem to need Prevnar 13 I believe that it is still given to certain high risk patients with CKD4-5, asplenia, cochlear implant etc. Are the pneumonia vaccines recs in vjs slide in line with new pneumonia vaccine recs Please see CDC website for details Nephrologist sometime use creatinine ratio, when do we check them Same data as microalbumincreatinine Optimal dosage of ACE-I ARB to decrease progression of CKD As high as the pt will tolerate without side effects Can you have CKD without albuminuria Yes, if they have other structural abnormalities- such as multiple cysts, a missing kidney etc. How about ace I or arbs in ESRd They can be used if needed for Rx HFrEF or hypertension At what point do diuretics stop working (GFR) Probably when pts stop making urine Do you still consider DM or HTN PT’s to have microalbuminuria even if the level normalizes with If they had repeated microalbuminuria, they would be considered to have CKD use of ACE OR ARB If these values improve with Rx, then they have CKD with improvment Are we supposed to be screening with annual microalbumincr ratio in HTN like we do with diabetes I believe that the ABFM hypertension part 4 module asks you to do this If you are comfortable doing so, you can treat with bicarbonate for CO2 < 22 That usually happens in late stage 3 CKD When do we treat with bicarb vs refer to renal They should see nephrology by the time they get to CKD 4 so one microalbuminuria does not mean diagnosis, it needs to be confirmed with a repeat test 2+tests3tests or 13 need to be positive Correct. Repeat positives indicate albuminuria so if the exam asks that a 33 yo with CKD comes to office, we need to give then PCV 13 then bring For pts with stage 4-5 CKD them back for ppV 23. How about Hep B series They should get hep B if not already immune How large are calcium deposits in skin with Calcipylaxis Not typically palpable, show up on x-ray Know JNC 8 for exam- 15090 for pts over 60, 14090 for everyone else 13080 for AHAACC can you clearly summarize the recommended blood pressure goals for each group Treat BP more agressively if albuminuria is present Our lab only reports up to GFR >60. Do some report up to >90 Our medical center lab reports estimated GFR from 0-120 For renal 2, know stages of CKD, main causes of CKD, main cause of death in CKD, use of ACE or ARB for CKD, hypertension, albuminuria, how to treat ACD, elements of secondary hyper parathyroidism, what NSF What are key points for the exam and calciphylaxis are in pt with CKD, their GFR can be fluctuant, do you keep them on dx of the worst stage or do you change the stage classificaiton everytime their GFR change Use their current baseline creatGFR in determining their stage, not the highest or lowest values for stage 1 what is the definition of kidney damage since GFR is normal These folks have persistent albuminuria or structural kidney disease seen on imaging Yes So do we stay with JNC-8 for meds and BP goals compared to the ACCAHA and KDOQI Unless asked specifically to do otherwise, the ABFM will most likely use JNC8 criteria and recommendations